Top Banner
www.Jspiders.com www.facebook.com/JSpiders.Basavanagudi/ Mb.9686114422 www.qspiders.com twitter.com/JBasavanagudi QUANTIATIVE APTITUDE 01. Basics 1. Solve -12+18-16-14-15+18+12 a) 9 b) -9 c) 10 d) 11 2. -16-15-14+25-18+16+38+27 a) 45 b) 44 c)43 d) 47 3. -2×3+4÷2-2×8÷4+5×2 a) 4 b) 6 c) 8 d) 2 4. -3×-8+4×-6+5×-3+4×-3 a) 25 b) -25 c) -27 d) 27 5. 1 2 + 3 2 8 6 + 5 12 9 8 = a) 1 24 b) 1 20 c) 1 24 d) 1 2 6. 3 5 × 5 16 3 8 × 12 18 + 2 7 × 14 16 = a) 1 2 b) 1 8 c) 1 8 d) 1 4 7. 3 5 ÷ 12 25 5 8 × 3 4 ÷ 15 16 2 3 = a) 1 2 b) 1 12 c) 5 2 d) 7 2 8. 2 3 [6 + 2 × 3 − 4(3 − 2 × 4 + 5) − 3] = a) 5 b) 8 c) 6 d) 7 9. 4 8 ( 2 3 + 5 6 )− 1 2 ÷ 3 4 = a) 1 2 b) 5 2 c) 1 12 d) 7 2 10. 3 − 4 − = −5 + 12 + 5 = a) 5 b) 3 c) 4 d) 6 11. 7 − 3(2 + 5) − 12 = 6(5 − 2) + 8 = a) 4 b) 6 c) 5 d) 7 12. 3 5 6 + 1 2 + 2 = 8 6 + 3 4 3 2 ℎ = a) 7 2 b) 9 28 c) 7 6 d) 10 21 13. 2 + 3 + 7 2 = −4 2 + 5 ℎ = a) 1 2 b) −3 2 c) 11 2 d) 10 7 14. 3(x-3)+4= 2(x-6)+6 then x= a) 2 b) 4 c) 3 d) 5 INFRINGE: BREAK LAW OR AGREEMENT, CONTRAVENE, TRANSGRESS, TRESPASS 02. Number System 1. 5793×999= a) 5787207 b) 4787307 c) 6787307 d) 4687307 2. 798×798−202 × 202 = a) 496000 b) 596000 c) 696000 d) 796000 3. The least value of *for which 5987*15 is divisible by 3 : a) 1 b)2 c) 3 d) 4 4. The least value of *for which 8*3464 is divisible by 9: a) 1 b)2 c) 3 d)4 5. Find the least value of * for which 4832*18 is divisible by 11 : a) 6 b)7 c) 8 d)9 6. On dividing a certain number by 342, we get 47 as remainder. If the same number is divided by 18, then remainder will be: a) 9 b) 10 c) 11 d) 12 7. On dividing a number by 56, we get 29 as remainder. On dividing the same number by 8 the remainder will be : a) 4 b) 5 c) 6 d) 7 8. The unit digit of the product 684×759×413×676 is a) 6 b) 8 c) 2 d) 4 9. The unit digit of the product (3547) 153 × (253) 72 is a) b) c) d) 10. How many prime factors are there in the product of [(4) 11 ×7 5 × (11) 2 a) 26 b)27 c) 28 d)29 11. What will be the remainder when 2 31 5: a) 2 b)1 c) 3 d)4 12. If the number 653 by 90 then (x +y) is: a) 2 b) 3 c) 4 d) 6 03. H.C.F AND L.C.M 1. Three numbers are in the ratio 1:2:3 and their HCF is 12. The numbers are:
37

JSpiders - Aptitude

Jan 22, 2017

Download

Software

Welcome message from author
This document is posted to help you gain knowledge. Please leave a comment to let me know what you think about it! Share it to your friends and learn new things together.
Transcript
Page 1: JSpiders - Aptitude

www.Jspiders.com www.facebook.com/JSpiders.Basavanagudi/ Mb.9686114422 www.qspiders.com twitter.com/JBasavanagudi

QUANTIATIVE APTITUDE

01. Basics

1. Solve -12+18-16-14-15+18+12

a) 9 b) -9

c) 10 d) 11

2. -16-15-14+25-18+16+38+27

a) 45 b) 44 c)43 d) 47

3. -2×3+4÷2-2×8÷4+5×2

a) 4 b) 6

c) 8 d) 2

4. -3×-8+4×-6+5×-3+4×-3

a) 25 b) -25

c) -27 d) 27

5. 1

2+

3

2−

8

6+

5

12−

9

8=

a) 1

24 b)

1

20

c) −1

24 d)

1

2

6. 3

5

16−

3

12

18+

2

14

16=

a) 1

2 b)

1

8

c) −1

8 d)

1

4

7. 3

12

25−

5

3

15

16−

2

3=

a) 1

2 b)

1

12

c) 5

2 d)

7

2

8. 2

3[6 + 2 × 3 − 4(3 − 2 × 4 + 5) − 3] =

a) 5 b) 8

c) 6 d) 7

9. 4

8(

2

3+

5

6) −

1

3

4=

a) 1

2 b)

5

2

c) 1

12 d)

7

2

10. 3𝑥 − 4 − 𝑥 = −5𝑥 + 12 + 5 =

a) 5 b) 3

c) 4 d) 6

11. 7𝑥 − 3(2𝑥 + 5) − 12 = 6(5 − 2𝑥) + 8 =

a) 4 b) 6

c) 5 d) 7

12. 𝑥

3−

5

6+

1

2+

𝑥

2=

8

6+

3

4−

3𝑥

2 𝑡ℎ𝑒𝑛 𝑥 =

a) 7

2 b)

9

28

c) 7

6 d)

10

21

13. 𝑥

2+

𝑥

3+

7

2=

𝑥−4

2+ 5 𝑡ℎ𝑒𝑛 𝑥 =

a) 1

2 b)

−3

2

c) 11

2 d)

10

7

14. 3(x-3)+4= 2(x-6)+6 then x=

a) 2 b) 4

c) 3 d) 5

INFRINGE: BREAK LAW OR AGREEMENT, CONTRAVENE,

TRANSGRESS, TRESPASS

02. Number System

1. 5793×999=

a) 5787207 b) 4787307

c) 6787307 d) 4687307

2. 798×798−202 × 202 =

a) 496000 b) 596000

c) 696000 d) 796000

3. The least value of *for which 5987*15 is divisible by 3

:

a) 1 b)2 c) 3 d) 4

4. The least value of *for which 8*3464 is divisible by 9:

a) 1 b)2

c) 3 d)4

5. Find the least value of * for which 4832*18 is divisible

by 11 :

a) 6 b)7

c) 8 d)9

6. On dividing a certain number by 342, we get 47 as

remainder. If the same number is divided by 18, then

remainder will be:

a) 9 b) 10

c) 11 d) 12

7. On dividing a number by 56, we get 29 as remainder.

On dividing the same number by 8 the remainder will

be :

a) 4 b) 5

c) 6 d) 7

8. The unit digit of the product 684×759×413×676 is

a) 6 b) 8

c) 2 d) 4

9. The unit digit of the product (3547)153 × (253)72 is

a) b)

c) d)

10. How many prime factors are there in the

product of [(4)11 × 75 × (11)2 ∶

a) 26 b)27

c) 28 d)29

11. What will be the remainder when

231 𝑖𝑠 𝑑𝑖𝑣𝑖𝑑𝑒𝑑 𝑏𝑦 5:

a) 2 b)1

c) 3 d)4

12. If the number 653𝑥𝑦 𝑖𝑠 𝑑𝑖𝑣𝑖𝑠𝑖𝑏𝑙𝑒 by 90 then

(x +y) is:

a) 2 b) 3

c) 4 d) 6

03. H.C.F AND L.C.M

1. Three numbers are in the ratio 1:2:3 and their HCF is

12. The numbers are:

Page 2: JSpiders - Aptitude

www.Jspiders.com www.facebook.com/JSpiders.Basavanagudi/ Mb.9686114422 www.qspiders.com twitter.com/JBasavanagudi

SALUTARY: BENEFICIAL, PRODUCTIVE, HEALTHY,

SALUBRIOUS

a) 4,8,12 b) 5,10,15

c) 10,20,30 d) 12,24,36

2. Two numbers are in the ratio 5:6.if HCF is 30 then

The sum of the two number is :

a) 50 b) 55

c) 60 d) 65

3. The HCF of two numbers is 11 and their LCM is 7700.

If one of the number is 275, then the other is :

a) 279 b) 283

c) 308 d) 318

4. The greatest number that exactly divides 105, 1001

and 2436 is :

a) 3 b) 7

c) 11 d) 21

5. Three different containers contain 496 litres, 403

litres and 713 litres of mixtures of milk and water

respectively. What biggest measure can measure all

the different quantities exactly:

a) 1 litre b) 7 litres

c) 31 litres d) 41 litres

6. The maximum number of students among them 1001

pens and 910 pencils can be distributed in such a way

that each students get same number of pens and

pencil:

a) 91 b) 910

c) 1001 d) 1911

7. Find the greatest number that will divide 43,91 and

183 so as to leave the same remainder in each case is

:

a) 4 b) 7

a) 9 d) 13

8. Find the greatest number that will divide

1356, 1868 and 2764 leaving the same

remainder in each case:

a) 64 b) 124

c) 156 d) 240

9. The greatest number which on dividing 1657

and 2037 leaves remainders 6 and 5

respectively, is :

a) 123 b) 127

c) 235 d) 305

10. Which of the following is a pair of co-

primes?

a) (16,62) b) (18,25)

c) (21, 35) d) (23, 92)

11. The least number which is exactly divisible

by 12, 15 and 18 is :

a) 160 b) 170

c) 180 d) 190

FUTILITY: POINTLESSNESS, VANITY,

HOLLOWNESS, FORLORNNESS

12. The least number which when divided by 12,

15, 20 and 54 leaves in each case a

remainder of 8 is:

a) 504 b) 536

c) 544 d) 548

13. Find the greatest number which on dividing

1657 and 2037 leaves remainders 6 and 5

respectively:

a) 125 b) 126

c) 127 d) 128

14. Find the least number exactly divisible by 12,

15, 20 and 27:

a) 540 b) 550

c) 560 d) 570

15. Find the least number which when divided

by 6, 7, 8, 9 and 12 leaves the same

remainder 1 in each case:

a) 504 b) 505

c) 506 d) 507

16. The LCM of 1

3,

5

6 ,

2

9 ,

4

27 𝑖𝑠:

a) 1

54 b)

10

27

c) 20

3 d) none

17. The HCF of 2

3 ,

8

9 ,

64

8 ,

10

27 𝑖𝑠

a) 2

3 b)

2

81

c) 160

3 d)

160

81

18. The traffic lights at three different road crossing

changes after every 48 secs. ,72 secs. and 108 secs.

respectively. If they all change simultaneously at

8:20:00 hours, then at what time will they again

change simultaneously:

a) 8:27:00 b) 8:27:08

c) 8:27:12 d) 8:27:15

19. Six bells commence ringing together and ring at

intervals 2, 4, 6 ,8,10 and 12 seconds respectively. in

40 minutes how many times do they ring together?

a) 18 b) 19

c) 20 d) 21

20. Four different electronic devices make a beep after

every 30 minutes, 1 hour, 11

2 ℎ𝑜𝑢𝑟 and 1 hour 45

minutes respectively. All the devices beeped together

at 12 noon they will again beep together at:

a) 12 midnight b) 3 a.m

c) 6 a.m d) 9 a.m

21. A, B and C start at the same time in the same

direction to run around a circular park. A completes a

round in 252 seconds, B in 308 seconds and C in 198

Page 3: JSpiders - Aptitude

www.Jspiders.com www.facebook.com/JSpiders.Basavanagudi/ Mb.9686114422 www.qspiders.com twitter.com/JBasavanagudi

seconds. All starting at the same point. After what

time will they all meet again at starting point?

REFUTE: PROVE TO BE WRONG, DENY, DISAPPROVE,

CONTROVERT, REBUT, DEBUNK, REPUDIATE

a) 26 minutes 18 seconds

b) 42 minutes 36 seconds

c) 45 minutes

d) 46 minutes 12 seconds

22. A man run from A to B with a speed of 20 km/hr and

comes back from B to A with a speed of 30 km/hr. His

avg. Speed of whole journey is :

a) 22 km/hr b) 24km/hr

c) 26 km/hr c) 28 km/hr

04. SIMPLIFICATION

1. What is x if 41

2+ 3

1

6+ 𝑥 + 2

1

3= 13

2

5

a) 41

2 b) 3

2

5

c) 43

5 d) 2

4

5

2. Find the value 4 −5

1+1

3+1

2+14

a) 1

5 b)

1

6

c) 1

7 d)

1

8

3. If 2𝑥

1+1

1+𝑥

1−𝑥

= 1 𝑡ℎ𝑒𝑛 𝑥 =

a) 2

3 b)

3

2

c) 4

3 d)

3

4

4. If 2x+3y=34 and 𝑥+𝑦

𝑦=

13

8 then 5y + 7x =

a) 65 b) 70

c) 75 d) 80

5. The value of (1 −1

3) (1 −

1

4) (1 −

1

5) … … (1 −

1

100) 𝑖𝑠

a) 1

20 b)

1

30

c) 1

50 d)

1

60

6. Value of 9948

49× 245 =

a) 25495 b) 24495

c) 26495 d) 30495

7. The value of 999995

999× 999 𝑖𝑠

a) 999996 b) 998986

c) 998996 d) 989996

8. A man divides 8600 among 5 sons, 4 daughters,

and 2 nephews. If each daughter receives four

times as much as each nephew, and each son

receives five times as much as each nephew, how

much does each daughter receives?

a) 700 b) 800

c) 900 d) 1000

PERSPICUOUS: CLEARLY EXPRESSED AND EASILY

UNDERSTOOD, LUCID, UNEQUIVOCAL

9. A man spends 2

5 𝑜𝑓 ℎ𝑖𝑠 𝑠𝑎𝑙𝑎𝑟𝑦 𝑜𝑛 ℎ𝑜𝑢𝑠𝑒 𝑟𝑒𝑛𝑡,

3

10 𝑜𝑓 ℎ𝑖𝑠 𝑠𝑎𝑙𝑎𝑟𝑦

on food and 1

8 of his salary on conveyance. If he

has 1400 left with him, then his expenditure on

food is :

a) 2000 b)2200

c) 2400 d) 2600

10. A third of rahul’s marks in mathematics exceeds

a half of his marks in english by 30. If he got 240

marks in the two subjects together, marks

obtained by him in english is :

a) 50 b) 60

c) 70 d) 80

11. If 1

8 𝑜𝑓 a pencil is black,

1

2 of the remaining is

white and the remaining 3 1

2 𝑐𝑚 is blue. Total

length of the pencil is :

a) 6cm b) 7cm

c) 8 cm d) 9cm

12. Mr. Rao is on a tourand he has 360 rs.for his

expences. If he exceeds his tour by 4 days, he

must cut down his daily expenses by 3 rs. Mr.

Rao is on tour for :

a) 15 days b) 16 days

c) 20 days d) 21 days

13. Two pens and three pencils cost rs. 86. Four pens

and a pencil cost rs. 112. Cost of a pen is :

a) 20 b) 24

c) 25 d) 30

14. Cost of three oranges and seven apples is 160 rs.

Then cost of fifteen oranges and thirty five

apples is :

a) 600 b) 500

c) 700 d) 800

15. Arun and Mohan are friends. Each has some

money. If Arun gives Rs. 30 to mohan then

mohan will have twice the money left with Arun.

But, if mohan gives Rs. 10 to Arun , then Arun will

have thrice as much as is left with Mohan. What

amount does arun has:

a) 60 b) 61

c) 62 d) 63

16. In a field , in addition to 50 hens there are 45

goats and 8 camels with some keepers. If the

Page 4: JSpiders - Aptitude

www.Jspiders.com www.facebook.com/JSpiders.Basavanagudi/ Mb.9686114422 www.qspiders.com twitter.com/JBasavanagudi

total number of feet be 224 more than the

number of heads , then number of kepers are :

a) 8 b) 10

c) 12 d) 15

HUNKY-DORY: COMPLETELY SATISFACTORY,

AGREEABLE, ALL RIGHT, COPACETIC, PALATABLE

17. A boy was asked to multiply a number by 25. He

instead multiplied the number by 52 and got the

answer 324 more than the correct answer. The

number to be multiplied was:

a) 12 b) 15

c) 25 d) 32

18. The total monthly salary of 4 men and 2 women

is Rs. 46000. If a woman eahrns Rs. 500 more

than a man, what is the monthly salary of a

woman?

a) 6500 b) 7500

c) 8000 d) 9000

19. A pineapple cost Rs. 7 each and a watermelon

cost Rs. 5 each . If Rahul spends Rs. 38 on these

fruits then number of pineapple purchased is:

a) 2 b) 3

c) 4 d) data inadequate

20. A sum of Rs. 1360 has been divided among A,B

and C such that A gets 2

3 of what B gets

And B gets 1

4 of what C gets. B’s share is:

a) 120 b) 160

c) 240 d) 300

21. A sum of Rs. 312 was divided among 100 boys

and girls in such a way that each boy gets Rs.

3.60 and each girls Rs. 2.40. the number of girls

is:

a) 35 b) 40

c) 60 d) 65

22. In an examination, a student score 4 marks for

every correct answer. If he attempts in all 60

questions and secures 130 marks, the no. of

questions he attempts correctly, is:

a) 35 b) 38

c) 40 d) 42

23. A cricket team won 3 matches more than they

lost. If a win gives them 2 points and loss (-1)

point, how many matches, in all have they played

if their score is 23?

a) 17 b) 20

c) 27 d) 40

24. The price of 10 chairs is equal to that of 4 tables .

the price of 15 chairs and 2 tables together is Rs.

4000. The total price of 12 chairs and 3 tables is :

a) 3500 b) 3750

c) 3840 d) 3900

25. In a group of cows and ducks, the number of legs

are 24 more than twice the number of heads.

Total no. of cows in the group is:

a) 6 b) 8

c) 10 d) 12

MUSTER: COME OR BRING TOGETHER ESPECIALLY FOR

INSPECTION, ASSEMBLE, MOBILIZE, CONVENE

05. SQUARE AND SQUARE ROOTS

1. √248 + √51 + √169 =

a) 12 b) 14

c) 16 d) 18

2. If √1 + 𝑥

144=

13

12, then x is

a) 15 b) 20

c) 25 d) 30

3. If x= 1+√2 and y= 1-√2 , the value of (𝑥2 + 𝑦2)

is:

a) 2 b) 6

c) 8 d) 10

4. The value of

√10 + √25 + √108 + √154 + √225 is

a) 4 b) 6

c) 8 c) 10

5. If 𝑥 = (7 − 4√3), 𝑡ℎ𝑒𝑛 𝑡ℎ𝑒 𝑣𝑎𝑙𝑢𝑒 𝑜𝑓 (x+ 1

𝑥) is:

a) 3√3 b) 8√3

c) 14 d) 14+8√3

6. A man plants 53824 apple trees in his garden

and arranges them so that there are as many

rows as there are apple trees in each row. No.

of rows is:

a) 231 b) 232

c) 233 d) 234

7. A general wishes to draw up his 8504 soilders

in the form of solid square. After arranging

them some are left over. How many are left:

a) 35 b) 40

c) 45 d) 50

06. AVERAGE

1. The average of first 40 natural numbers is:

a) 20 b) 20.5

c) 21 d) 22

2. The average of four consecutive even numbers is

27. The largest number is :

a) 28 b) 29

Page 5: JSpiders - Aptitude

www.Jspiders.com www.facebook.com/JSpiders.Basavanagudi/ Mb.9686114422 www.qspiders.com twitter.com/JBasavanagudi

c) 30 d) 31

3. The average of first nine prime numbers is:

a) 9 b) 11

c) 11 1

9 d) 11

2

9

4. If Arithmetic mean of 3, 11, 7, 9, 15, 13, x, 8, 19,

17, 21, 14 is 12 then x is :

a) 3 b) 7

c) 17 d) 31

CONSENSUS: GENERAL AGREEMENT, ACCORD,

UNITY, SOLIDARITY

5. If the average of 2, 7, 6 and x is 5 and the average

of 18, 1, 6, x and y is 10. What is the value of y?

a) 5 b) 10

c) 20 d) 30

6. If the mean of 5 observations x, x+2, x+4, x+6 and

x+8 is 11, then the mean of the last three

observations is:

a) 11 b) 13

c) 15 d) 17

7. The average of first 50 natural numbers is :

a) 12.25 b) 21.25

c) 25 d) 25.5

8. The average age of 35 students in a class is 16

years. The average age of 21 students is 14. What

is the average age of remaining 14 stubents?

a) 15 years b) 17 years

c) 18 years d) 19 years

9. The average of runs of a cricket player of 10

innings was 32. How many runs must he make in

his next inning so as to increase his average of

runs by 4/

a) 2 b) 4

c) 70 d) 76

10. A grocer has a sale of Rs. 6435, 6927, 6855, 7230

and 6562 for consecutive 5 months. How much

sale must he have in the sixth month so that he

gets an average sale of Rs. 6500.

a) 4991 b) 5991

c) 6001 d) 6991

11. Out of 9 persons , 8 persons spent Rs. 30 each for

their meals. The ninth one spent 20 more than

the average expenditure of all the nine. The total

money spent by all of them was:

a) 260 b) 290

c) 292.50 d) 400.50

12. The mean of 50 observations was 36. It was

found later that an observation 48 was wrongly

taken as 23. The corrected new mean is:

a) 35.2 b) 36.1

c) 36.5 d) 39.1

13. The average age of 15 students in a class is 15

years. Out of these, the average age of 5

students is 14 years and that of the other 9 is 16

years. The age of the 15th student is:

a) 11 years b) 14 years

c) 15 yerars d) 15 2

7 years

14. The average of 11 numbers is 10.9. if the average

of the first six numbers is 10.5 and that of last six

is 11.4, then the middle number is :

a) 11 b) 11.3

c) 11.4 d) 11.5

PETULANT: CHILDISHLY SULKY, BAD TEMPERED,

PEEVISH

15. The avg. weight of A,B and C is 45 kg. if the avg.

weight of A and B be 40 kg and that of B and C be

43 kg., then the weight of B is:

A) 17 kg b) 20 kg

C) 26kg d) 31kg

16. The avg. age of 36 students in a class is 14 years.

When teacher’s age is included to it, the average

increases by one. What is the teacher’s age?

a) 31 years b) 36 years

c) 51 years d) 60 year

17. The average age of 15 persons increases by 1.2

years when one of the person who weighs 53kg

is replaced by a new man. Weight of the new

man is :

a) 65kg b) 70kg

c) 71kg d) 75kg

18. A train move from point A to point B with a

speed of 50km/hr and returns to point A with 40

km/hr. the avg. speed of the whole journey is:

a) 45 4

9 km/hr b) 44

4

9 km/hr

c) 50km/hr d) 60km/hr

07. PROBLEM ON NUMBERS

1. A number is as much greater than 36 as is less

than 86. Then the number is :

a) 60 b) 61

c) 62 d) 63

2. The sum of a rational number and its reciprocal is 13

6. Then the number is:

a) 2

3 b)

1

8

c) 5

8 d)

5

6

3. The sum of squares of three consecutive odd

numbers is 83. The largest number is

a) 5 b) 9

c) 7 d) 11

Page 6: JSpiders - Aptitude

www.Jspiders.com www.facebook.com/JSpiders.Basavanagudi/ Mb.9686114422 www.qspiders.com twitter.com/JBasavanagudi

4. 50 is divided into two parts such that the sum of

their reciprocal is 1

12. The two parts are :

a) 40, 10 b) 45, 5

c) 30,20 d) 32, 18

5. A fraction becomes 2

3 when 1 is added to both,

its numerator and denominator . and it becomes 1

2 when 1 is substracted from both numerator

and denominator. The fraction is:

a) 1

8 b)

3

8

c) 4

5 d)

3

5

SERENDIPITY: GOOD LUCK, FORTUNE, PROVIDENCE,

ACCIDENTAL, COINCIDENTAL

6. If the sum of one-half and one-fifth of a number

exceeds one-third of that number by 7 1

3 , the

number is :

a) 15 b) 18

c) 20 d) 30

7. Twenty times a positive integer is less than its

square by 96. Then the number is:

a) 20 b) 24

c) 30 d) 50

8. The ratio between two numbers is 4:7. If each is

increased by 4, the ratio becomes 3:5. The larger

number is :

a) 36 b) 48

c) 56 d) 64

9. The sum and difference of two numbers are 20

and 8 respectively, then difference of their

squares is :

a) 12 b) 28

c) 160 d) 180

10. The sum of four consecutive even integers is

1284. The largest of them is :

a) 320 b) 322

c) 324 d) 326

08. PROBLEMS ON AGES

1. Rahul’s age after 15 years will be 5 times his age

5 years back. The presesnt age of rahul is :

a) 8 years b) 10 years

c) 12 years d) 14 years

2. The present age of father is 3 years more than

three times the age of his son. Three years

hence, father’s age will be 10 years more than

twice the age of the son. Present age of father is :

a) 30 years b) 32 years

c) 33 years d) 35 years

3. One year ago, the ratio of gaurav’s and sachin’s

age was 6:7 respectively. Four years hence the

ratio would become 7:8. The age of sachin is :

a) 32 years b) 34 years

c) 38 years d) 36 years

4. Present age of Sameer and Anand are in the ratio

5:4 respectively. Three years hence the ratio of

their ages will become 11:9 respectively. Anand’s

present age is:

a) 24 years b) 27 years

c) 40 years d) 42 years

5. Ten years ago A was half of B in age. If the ratio

of their present ages is 3:4, then total of their

present ages :

IMPETUOUS: ACTING OR DONE QUICKLY OR RASHLY,

IMPULSIVE AND FORCEFUL, INJUDICIOUS

a) 20 years b) 30 years

c) 45 years d) none

6. The sum of present ages of a father and his son is

60 years. Six years ago, father’s age was five

times the the age of the son. After 6 years son’s

age will be :

a) 12 years b) 14 years

c) 18 years d) 20 years

7. The sum of ages of 5 children born at the

intervals of 3 years each is 50 years. What is the

age of the youngest child:

a) 4 years b) 8 years

c) 10 years d) none

09. PERCENTAGE

1. 28% of 450 + 45% of 280 is

a) 150 b) 252

c) 352 d) 450

2. What percent of 6 is 90 ?

a) 1200% b) 1300%

c) 1400% d) 1500%

3. 9% of what is 63?

a) 70 b) 50

c) 60 d) 90

4. If the sales tax be reduced from 31

2% 𝑡𝑜 3

1

3% ,

then what difference does it make to a person

who purchases an article with marked price of

7200?

a) 9 b) 15

c) 18 d) 12

5. 60% of 264 is same as :

a) 10% of 44 b) 15% of 1056

c) 30% of 132 d) none

Page 7: JSpiders - Aptitude

www.Jspiders.com www.facebook.com/JSpiders.Basavanagudi/ Mb.9686114422 www.qspiders.com twitter.com/JBasavanagudi

6. 270 candidates appeared for the examination, of

which 252 passed. The pass percentage is :

a) 80% b) 831

2%

c) 901

3% d) 93

1

3%

7. In an election a candidate who gets 84% of the

votes is elected by a majority of 476 votes. The

total no. of votes polled is:

a) 672 b) 700

c) 749 d) 848

8. If p% of p is 36, then p is equal to:

a) 15 b) 60

c) 600 d) 3600

9. Rohith spend 40% of his salary on food, 20% on

house rent, 10% on entertainment and 10% on

conveyance. If his savings at the end of a month

are Rs. 1500, then his monthly salary is:

TUMULTUOUS: VERY LOUD OR UPROARIOUS, EXCITED,

CONFUSED, VOCIFEROUS, BOISTEROUS, FRENZIED,

RUMBUSTIOUS

a) 6000 b) 7500

c) 8000 d) 10,000

10. 405 sweets are equally distributed among children

in such a way that the number of sweets received

by each child is 20% of the total number of

children. Sweet received by each child is:

a) 9 b) 15

c) 18 d) 45

11. What % decrease in salaries would exactly cancel

out the 20 percent increase:

a) 162

3 b) 18

c) 20 d) 331

3

12. A number is increased by 20% and then again by

20%. By what % should the increased salary be

decreased so as to get back the original number :

a) 1911

13 b) 30

5

9

c) 40% d) 44

13. The price of tea being increased by 20%, a man

reduces his consumption by 20%. By how much

percent will his expenses for tea be decreased?

a) 2% b) 4%

c) 6% d) 8%

14. If the price of petrol is increased by 30% , by how

much percent a car owner must reduce his

consumption in order to maintain same budget:

a) 21% b) 211

3%

c) 231

13% d) 33%

15. The price of wheat falls by 16%. By what

percentage a person can increase the

consumption of wheat so his overall budget does

not change:

a) 16% b) 18%

c) 18.5% d) 19%

16. The population of a town 2 years ago was 62,500.

Due to migration of big cities, it decreases every

year at a rate of 4% . the present population of

the town is:

a) 56,700 b) 57,600

c) 58800 d) 60,000

17. The value of machine depreciates at the rate of

10%. It was purchased 3 years ago. If its present

value is 8748, its purchase price was:

a) 10,000 b) 11,372

c) 12,000 d) 12,500

18. A’s salary is 50% more than B’s. how much

percent is B’s salary less than A’s:

a) 33% b) 331

3%

c) 331

4% d) 33

1

2%

APLOMB: SELF-CONFIDENCE, POISE, COMPOSURE,

SANGFROID, EQUILIBRIUM, EQUANIMITY

19. In an election between two candidates, 75% of

the voters cast their votes, out of which 2% of the

voters were declared invalid. A candidate got

9261 votes which were 75% of the total valid

votes. Then total no of votes enrolled in that

election:

a) 16000 b) 18000

c) 16800 d) 17000

20. Raman’s salary was decreased 50% and

subsequently increased by 50%. What % does he

lose?

a) 10% b) 20%

c) 25% d) 30%

21. The salary of a person was reduced by 10%. By

what % should his reduced salary be raised so as

to bring it at par with his original salary:

a) 12% b) 10%

c) 111

9% d) 11%

22. If the numerator of a fraction be increased by 15%

and its denominator be diminished by 8%, the

value of the fraction is 15

16 . find the original

fraction:

a) 3

4 b)

1

2

c) 5

4 d)

6

7

23. The population of a town is 1, 76,400. If it

increases at the rate of 5% per annum, what was

its population 2 years ago:

a) 12000 b) 15000

c) 14000 d) 16000

Page 8: JSpiders - Aptitude

www.Jspiders.com www.facebook.com/JSpiders.Basavanagudi/ Mb.9686114422 www.qspiders.com twitter.com/JBasavanagudi

24. If A earns 331

3% more than B, how much percent

does B earn less than A?

a) 20% b) 24%

c) 25% d) 28%

25. If A’s salary is 20% less than B’s salary, by how

much percent is B’s salary more than A’s:

a) 20% b) 24%

c) 25% d) 23%

10. PROFIT AND LOSS

1. If a radio is purchased for Rs. 490 and sold for Rs.

465.50, then loss percent is :

a) 6% b) 4%

c) 5% d) 10%

2. What is S.P when C.P is 540 and profit is 10%:

a) 494 b) 594

c) 600 d) 650

3. What is C.P when S.P is 40.60 and gain is 16%:

a) 30 b) 34

c) 35 d) 32

MUNIFICENT: VERY GENEROUS, BOUNTIFUL, BIG-

HEARTED, FULSOME, UNSTINTING, UNSPARING

4. Alfred buys an old scooter for Rs. 4700 and

spends Rs. 800 on its repairs. if he sells the

scooter for Rs. 5800, his gain percent is :

a) 44

7% b) 5

5

11%

c) 10% d) 12%

5. 100 oranges are bought at the rate of Rs. 350 and

sold at the rate of Rs. 48 per dozen.The

percentage of profit or loss is:

EXCRUCIATING: intensely painful, very

embarrassing, agonizing, intense

a) 142

7% gain b) 15% gain

c) 142

7% loss d) 15% loss

6. Peter purchased a machine for Rs. 80,000 and

spent Rs. 5000 on repair and Rs. 1000 on

transport and sold it with 25% profit. At what

price did he sell the machine :

a) 1,05,100 b) 1,06,250

c) 1,07, 500 d) 1, 17, 500

7. When a plot is sold for 18,700, the owner loses

15%. At what price must the plot be sold in order

to gain 15%:

a) 21,000 b) 22,500

c) 25,300 d) 25,800

8. The S.P of an article is 4

3 of its C.P, the profit in the

transaction is :

a) 162

3% b) 20

1

2%

c) 251

2% d) 33

1

3%

9. The ratio of C.P and S.P is 4:5. The profit percent

is :

a) 10% b) 20%

c) 30% d) 25%

10. If the cost price of 12 pens is equal to the S.P of 8

pens, the gain % is:

a) 25% b) 331

3%

c) 50% d) 662

3%

11. The CP of 19 articles is equal to the selling price

of 16 articles. Gain % is:

a) 39

17% b) 15

15

19%

c) 183

4% d) 20%

12. Oranges are bought at the rate of 10 for Rs. 25

and sold at the rate of 9 for Rs. 25. The profit %

is:

a) 91

11% b) 10%

c) 111

9% d) 12

1

2%

13. Some articles were bought at 6 for Rs. 5 and sold

at 5 for Rs. 6. Gain% is:

a) 30% b) 331

3%

c) 35% d) 44%

TROGLODYTE: A PERSON WHO IS OLD DELIBERATELY

IGNORANT OR OLD FASHIONED

14. A bag marked at Rs. 80 is sold for Rs. 68. The rate

of discount is :

a) 12% b) 15%

c) 16% d) 20%

15. A fan is listed at Rs 1500 and a discount of 20% is

offered on the list price. What additional

discount must be offered to the customer to

bring the net price to Rs. 1104:

a) 8% b) 10%

c) 12% d) 15%

16. List price of an article is 2000 and it is being sold

at successive discount of 20% and 10%. Its selling

price will be:

a) 1400 b) 1440

c) 1520 d) 1700

17. After successive discounts of 12% and 5% an

article was sold for Rs. 209. What was the

original price of the article:

a) 226 b) 250

c) 252 d) 269

18. The marked price of a watch was Rs. 720. A man

bought the same for Rs. 550.80 after getting two

successive discounts, the first being 10%. What

was the second discount:

a) 12% b) 14%

c) 15% d) 18%

Page 9: JSpiders - Aptitude

www.Jspiders.com www.facebook.com/JSpiders.Basavanagudi/ Mb.9686114422 www.qspiders.com twitter.com/JBasavanagudi

19. A tradesman marked his goods 30% above the

CP. If he allows a discount of 10%, then his gain %

is:

a) 15% b) 16%

c) 17% d) 18%

20. A shopkeeper fixed the marked price of his

article 35% above its cost price. The percentage

of discount allowed to gain 8% is:

a) 20% b) 27%

c) 31% d) 43%

21. A person loses 5% by selling a watch for Rs.

1140. At what price should the watch be sold to

earn 5% profit:

a) 1200 b) 1260

c) 1300 d) 1400

22. A vendor bought bananas at 6 for Rs. 10 and sold

them at 4 for Rs. 6. Loss % is:

a) 5% b) 10%

c) 12% d) 14%

23. The manufacturer gains 10%, the wholesale

dealer 15% and the retailer 25%. If its retail price

is 1265 then cost of production of table is:

a) 700 b) 800

c) 900 d) 600

24. Monika purchased a pressure cooker at9

10𝑡ℎ of its SP

and sold it at 8% more than its SP. Her gain % is :

MOUNTBANK: A PERSON WHO DECEIVES OTHERS,

CHARLATAN, IMPOSTER, QUACK

a) 16% b) 18%

c) 25% d) 20%

25. The price of jewel, passing through three hands,

rises on the whole 65%. If the first and second

sellers earned 20% and 25% profit respectively.

Then profit earned by third seller is :

a) 12% b) 10%

c) 15% d) 16%

26. A man bought a horse and a carriage for Rs.

3000. He sold the horse at a gain of 20% and the

carriage at a loss of 10%, thereby gaining 2% on

the whole. Then CP of the horse is:

a) 1000 b) 1200

c) 1500 d) 1400

27. Find the equivalent single discount to a series

discount of 20% and 10%:

a) 25% b) 26%

c) 27% d) 28%

28. Find the equivalent discount to a series of

discount 20%, 10% and 5% :

a) 25% b) 30%

c) 31.6% d) 32%

29. An uneducated retailer marks all his goods at

50% above the cost price and thinking that he

will still make 25% profit, offers 25% discount on

the marked price. His actual profit would be :

a) 25% b) 20%

c) 12.50% d) 15%

11. RATIO AND PROPORTION

1. If a:b= 5:9 and b:c is 4:7, then a:b:c is

a) 5:9:7 b) 5:4:7

c) 20:36:63 d) 20:30:63

2. 2. If A:B= 2:3, B:C is 4:5, C:D is 6:7 then A:B:C:D is :

a) 16:22:30:35 b) 16:24:15:35

c) 16:24:30:35 d) 18:24:30:35

3. 3. If A:B =8:15, B:C= 5:8, C:D =4:5 then A:D =

a) 2:7 b) 4:15

c) 8:15 d) 15:4

4. 4. If A:B = 1

2:

3

8, B:C=

1

3:

5

9, C:D=

5

6:

3

4, then the ratio of A:B:C:D is:

a) 4:6:8:10 b) 6:4:8:10

c) 6:8:9:10 d) 8:6:10:9

5. 5. If 1

5:

1

𝑥=

1

𝑥:

1

1.25, then the value of x is:

a) 1.5 b) 2

c) 2.5 d) 3.5

6. 6. 0.75: 𝑥 ∷ 5: 8, then x is equal to:

a) 1.12 b) 1.20

c) 1.25 d) 1.30

7. 7. Fourth proportional of 4,9 and 12 is:

a) 22 b) 24

c) 26 d) 27

EMBRYONIC: RUDIMENTARY, UNDEVELOPED, IMMATURE,

INCHOATE, NASCENT, GERMINAL

8. The third proportional of 16 and 36 is:

a) 79 b) 80

c) 81 d) 82

9. The mean proportional of 144 and 225 is:

a) 160 b) 170

c) 180 d) 181

10. 10. Divide 672 in the ratio 5:3 ratio. Two parts are :

a) 420, 252 b) 440, 250

c) 450, 252 d) 420, 250

11. 11. Rs. 1162 is divided among A,B and C in the ratio 35:28:20.

amount received by A is :

a) 450 b) 460

c) 470 d) 490

12. 12. A bag contains 50p, 25p and 10p coins in the ratio 5:9:4,

amounting to Rs. 206. Then the number of coins of each type is

:

a) 200, 360 and 160 b) 200, 160,360

c) 200, 350, 120 d) 180, 120 and 360

13. 13. A mixture contains alcohol and water in the ratio 4:3. If 5

litres of water is added to the mixture, the mixture ratio

becomes 4:5. Then the quantity of alcohol in the mixture is:

a) 8 litres b) 10 litres

Page 10: JSpiders - Aptitude

www.Jspiders.com www.facebook.com/JSpiders.Basavanagudi/ Mb.9686114422 www.qspiders.com twitter.com/JBasavanagudi

c) 12 litres d) 14 litres

14. 782 is divided into three parts in the ratio 1

2:

2

3:

3

4, then

the first part is:

a) 182 b) 190

c) 196 d) 204

15. In a bag there are coins of 25P, 10p and 5p in the ratio

1:2:3. If there are Rs. 30 in all, how many 5p coins are

there :

a) 50 b) 100

c) 150 d) 200

16. 15 litres of mixture contains 20% alcohol and rest

water. if 3 litres of water is mixed with it, the

percentage of alcohol in the new mixture would be :

a) 15% b) 162

3%

b) 17% d) 181

2%

12. PARTNERSHIP

1. A, B and C started a business by investing Rs. 1,20,000,

Rs. 1,35,000 and Rs. 1,50,000 respectively. Find the

share of A, out of an annual profit of Rs. 56,700:

a) 16000 b) 16,800

c) 17000 d) 17,500

2. Alfred started a business investing Rs. 45,000. After 3

months peter joined him with a capital of Rs. 60,000.

After another 6 months, Ronald joined them with a

capital of Rs. 90,000. At the end of the year, they

made a profit of 16,500. The share of peter is:

VENERABLE: ACCORDED GREAT RESPECT BECAUSE OF

AGE, WISDOM OR CHARACTER, REVERED, ESTEEMED,

HALLOWEED

a) 6500 b) 6800

c) 6700 d) 6600

3. A,B and C start a business each investing Rs. 20,000.

After 5 months A withdrew Rs. 5000, B withdrew Rs.

4000 and C invests Rs. 6000 more. At the end of the

year, a total profit of Rs. 69,900 was recorded. Share

of C is:

a) 28000 b) 28200

c) 29000 d) 29000

4. Reena and shaloo are partners in a business. Reena

invested Rs. 35000 for 8 months and shaloo invests Rs. 42000

for 10 months. Out of a profit of Rs. 31,570, reena’s share is :

a) 9471 b) 12,628

c) 18,040 d) 18,942

13. CHAIN RULE

1. If 15 toys cost Rs. 234, what do 35 toys cost:

a) 540 b) 550

c) 560 d) 570

2. If 36 men can do a piece of work in 25 hours, how many

hours will 15 men take to do it:

a) 40 hours b) 50 hours

c) 60 hours d) 70 hours

3. If the wages of 6 men for 15 days to be Rs. 2100, then

wages of 9 men for 12 days :

a) 2500 b) 2520

c) 2600 d) 2700

4. If 20 men can build a wall 56 meters long in 6 days, what

length of a similar wall can be built by 35 men in 3 days :

a) 50m b) 49m

c) 45m d) 48m

5. If 15 men working 9 hours a day, can reap a field in 16 days,

in how many days will 18 men reap the field, working 8

hours a day:

a) 12 b) 14

c) 15 d) 16

6. 36 men can complete a piece of work in 18 days. In how

many days will 27 men complete the work:

a) 12 b) 18

c) 22 d) 24

7. A fort had provision of food for 150 men for 45 days. After

10 days, 25 men left the fort. The number of days for

which the remaining food will last is :

a) 291

5 b) 37

1

4

c) 42 d) 54

8. 12 men working 8 hours per day completes a piece of

work in 10 days. To complete the same work in 8 days,

working 15 hours a day , the number of men required is:

a) 4 b) 5

c) 6 d) 8

CARTE BLANCHE: COMPLETE FREEDOM TO ACT AS ONE

WISHES OR THINKS BEST

9. 10 men working 6 hours a day can complete a work in

18 days. How many hours a day must 15 men work to

complete the same work in 12 days:

a) 6 b) 10

c) 12 d) 15

10. In a camp, 95 men had provision for 200 days. After 5

days, 30 men left the camp. For how many days will

the remaining food last:

a) 180 b) 285

c) 13916

19 d) none

11. A garrison of 500 men had provision for 27 days. After

3 days a reinforcement of 300 men arrived. For how

many more days will the remaining food last now:

a) 15 b) 16

c) 171

2 d) 18

14. TIME AND WORK

Page 11: JSpiders - Aptitude

www.Jspiders.com www.facebook.com/JSpiders.Basavanagudi/ Mb.9686114422 www.qspiders.com twitter.com/JBasavanagudi

1. Worker A takes 8 hours to do a job. Worker B

takes 10 hours to do the same job. How long

should it take both A and B, working together but

independently, to do the same job?

a) 44

9 days b) 5

4

9 days

c) 6 days d) 7 days

2. A and B together can complete a piece of work in

4 days. If A alone can complete the same work in

12 days, in how many days can B alone complete

the work?

a) 4 days b) 5 days

c) 6 days d) 7 days

3. A does a work in 10 days and B does the same work in

15 days. In how many days they together do the same

work?

a) 5 days b) 6 days

c) 8 days d) 9 days

4. A can finish the work in 18 days and B can do the same

work in half the time taken by A. Then working together,

what part of the same work they can finish in a day?

a) 1

6 b)

1

9

c) 2

5 d)

2

7

5. A, B and C can complete the work in 24, 6 and 12 days

respectively. Working together they will complete the

same work in:

a) 1

24 𝑑𝑎𝑦𝑠 b)

7

24 𝑑𝑎𝑦𝑠

c) 33

7 𝑑𝑎𝑦𝑠 c) 4 days

CYNOSURE: A PERSON OR THING THAT IS CENTRE OF

ATTENTION OR ADMIRATION

6. A man can do a job in 15 days. His father takes 20 days

and his son finishes it in 25 days. How long will they take

to complete the job if they all work together:

a) Less than 6 days

b) exactly 6 days

c) Approximately 6.4 days

d) more than 10 days

7. A man can do a piece of work in 5 days, but with the help

of his son, he can do it in 3 days. In what time can the son

do it alone:

a) 6 1

2 days b) 7 days

c) 71

2 days d) 8 days

8. A can lay a railway line in 16 days and B can do the same

job in 12 days. With the help of C, they did the job in 4

days only. Then C alone can do the job in:

a) 91

5 𝑑𝑎𝑦𝑠 b)9

2

5 days

c) 93

5 𝑑𝑎𝑦𝑠 d) 10 days

9. A takes twice as much time as B and thrice as much time

as C to finish a piece of work. Working together they can

finish the work in 2 days. B alone can do the work in:

a) 4 days b) 6 days

c) 8 days d) 12 days

10. A and B can do a work in 12 days, B and C in 15 days, C

and A in 20 days. If A,B and C work together , they will

complete the work in:

a) 5 days b) 75

6 𝑑𝑎𝑦𝑠

c) 10 days d) 152

3 𝑑𝑎𝑦𝑠

11. A and B can do a work in 12 days, B and C in 15

days, C and A in 20 days. If A, B and C work together, they

will complete the work in:

a) 5 days b) 75

6 𝑑𝑎𝑦𝑠

c) 10 days d) 152

3 𝑑𝑎𝑦𝑠

12. A and B can do a piece of work in 72 days, B and C can

do it in 120 days; A and C can do it in 90 days. In what

time can A alone do it:

a) 80 days b) 100 days

c) 120 days d) 150 days

13. A and B can do a piece of work in 5 days; B and C can

do it in 7 days; A and C can do it in 4 days. Who

among these will take the least time if put to do it

alone?

a) A b) B

c) C d) data inadequate

14. A is twice as good a workman as B and together

they take 14 days to finish the work. The number of days

taken by A alone to finish the work is :

a) 11 b) 21

c) 28 d) 42

CONUNDRUM: A CONFUSING AND DIFFICULT PROBLEM,

MYSTERY, ENIGMA, PUZZLE

15. A can do a work in 15 days and B in 20 days. If

they work on it together for 4 days, then the fraction of

the work that is left is:

a) 1

4 b)

1

10

c) 7

15 d)

8

15

16. A can finish a work in 18 days and B can do the

same work in 15 days. B worked for 10 days and left the

job. In how many days, A alone can finish the remaining

work?

a) 5 b) 51

2

c) 6 d) 8

17. 10 men can complete a piece of work in 15 days

and 15 women can complete the same work in 12 days. If

all the 10 men and 15 women work together, in how many

days will the work get completed?

Page 12: JSpiders - Aptitude

www.Jspiders.com www.facebook.com/JSpiders.Basavanagudi/ Mb.9686114422 www.qspiders.com twitter.com/JBasavanagudi

a) 6 b) 61

3

c) 62

3 d) 7

2

3

18. A is twice as good a workman as B and together they

finish a piece of work in 18 days. In how many days will A

alone finish the work?

a) 25 days b) 27 days

c) 28 days d) 29 days

19. 2 men and 3 boys can do a piece of work in 10 days while 3

men and 2 boys can do the same work in 8 days. In how

many days can 2 men and 1 boy do the work?

a) 12 days b) 121

2 days

c) 13 days d) 131

2 days

15. PIPES AND CISTERNS

1. Two pipes A and B can fill a tank in 36 hours and 45 hours

respectively. If both the pipes are opened simultaneously,

how much time will be taken to fill the tank?

a. 15 hours b. 20 hours

c. 25 hours d. 30 hours

2. Two pipes can fill a tank in 10 hours and 12 hours

respectively while a third pipe empties the full tank in 20

hours. If all the three pipes operate simultaneously, in how

much time will the tank be filled?

a. 6 hours 10 mins

b. 7 hours 20 mins.

c. 7 hours 30 mins

d. 8 hours 20 mins.

3. If two pipes function simultaneously, the reservoir will be

filled in 12 hours. One pipe fills the reservoir 10 hours

faster than the other. How many hours does it take the

second pipe to fill the reservoir?

SUCCOUR: ASSISTANCE AND SUPPORT IN TIMES OF

HARDSHIP AND DISTRESS, HELP

a. 20 hours

b. 30 hours

c. 40 hours

d. 45 hours

4. An electric pump can fill a tank in 3 hours. Because of a

leak in the tank, it took 31

2 hours to fill the tank. If the tank

is full, how much time will the leak take to empty it:

a) 20 hours b) 21 hours

c) 22 hours d) 24 hours

5. Two pipes A and B can fill a tank in 20 and 30 mins

respectively. If both the pipes are used together, then how

long will it take to fill the tank:

a) 12 mins b) 15 mins

c) 25 mins d) 50 mins

6. A cistern can be filled by a tap in 4 hours while it can be

emptied by other tap in 9 hours. If both the taps are

opened simultaneously, then after how much time will the

cistern get filled?

a) 4.5 hours b) 5 hours

c) 6.5 hours d) 7.2 hours

7. Pipe A can fill a tank in 5 hours, pipe B in 10 hours and pipe

C in 30 hours. If all the pipes are opened, in how many

hours will the tank be filled :

a) 2 b) 2.5

c) 3 d) 3.5

8. Pipe A and B can fill a tank in 5 and 6 hours respectively.

Pipe C can empty it in 12 hours. If all the three pipes are

opened together, then the tank will be filled in:

a) 113

17 ℎ𝑜𝑢𝑟𝑠 b) 2

8

11 ℎ𝑜𝑢𝑟𝑠

c) 39

17 hours d) 4

1

2 ℎ𝑜𝑢𝑟𝑠

9. A pump can fill a tank in 2 hours. Because of a leak, it took

21

3 hours to fill the tank. The leak can drain all the water of

a tank in:

a) 41

3 ℎ𝑜𝑢𝑟𝑠 b) 7 hours

c) 8 hours d) 14 hours

10. One pipe can fill a tank three times as fast as another pipe.

If together the two pipes can fill the tank in 36 minutes,

then time taken by slower pipe to fill the tank in:

a) 81 minutes b) 108 mins

c) 144 mins d) 192 mins

16. TIME AND DISTANCE

1. A cyclist covers a distance of 750 m in 2 mins 30 secs.

What is the speed in km/hr of the cyclist:

a) 15 km/hr b) 17km/hr

c) 18 km/hr d) 20 km/hr

2. Peter can cover a certain distance in 1 hr. 24 min. by

covering two third of the distance at 4 kmph and the rest

at 5kmph . total distance of of the journey is:

ELUCIDATE: MAKE CLEAR ,EXPLAIN, UNRAVEL, UNRIDDLE

a) 5 km b) 6km

c) 7km d) 8km

3. Walking at 5

6 𝑜𝑓 𝑖ts usual speed, a train is 10 minutes too

late. Then its usual time to cover the journey is:

a) 40 mins b) 50 mins

c) 30 mins d) 45 mins

4. Which of the following trains is the fastest?

a) 25m/sec b) 1500m/min

c) 90km/hr d) none

5. A man walking at the rate of 5km/hr crosses a bridge in 15

minutes. The length of the bridge is:

a) 600 b) 750

c) 1000 d) 1250

Page 13: JSpiders - Aptitude

www.Jspiders.com www.facebook.com/JSpiders.Basavanagudi/ Mb.9686114422 www.qspiders.com twitter.com/JBasavanagudi

6. How long will a boy take to run round a square field of side

35 metres, if he runs at a rate of 9km/hr?

a) 50 secs b) 52 secs

c) 54 secs d) 56 secs

7. A certain distance is covered by a cyclist at a certain speed.

If jogger covers half the distance in double the time, the

ratio of the speed of the jogger to that of the cyclist is:

a) 1:2 b) 2:1

c) 1:4 d) 4:1

8. A can completes a journey in 10 hours. He travels first half

of the journey at the rate of 21km/hr and second half at

the rate of 24 km/hr. the total journey in km:

a) 220km b) 224km

c) 230km d) 234km

9. A person travels equal distance with 3km/hr, 4km/hr and

5km/hr and takes a total time of 47 mins. The total

distance is:

a) 2km b) 3km

c) 4km d) 5km

10. A train running at 7

11 of its own speed reached a place in

22 hours. How much time could be saved if the train would

have run at its own speed:

a) 7 hours b) 8 hours

c) 14 hours d) 16 hours

11. Walking 6

7 of his usual speed a man is 12 minutes too late.

The usual time taken by him to cover that distance is:

a) 1 hour b) 1 hour 12 min.

c) 1 hour 15 min. d) 1 hour 20 min.

12. Three persons are walking from a place A to another place

B. their speeds are in the ratio 4:3:5. The time ratio to

reach B by these persons will be :

a) 4:3:5 b) 5:3:4

c) 15:9:20 d) 15:20:12

13. A is twice as fast as B and B is thrice as fast as C is. The

journey covered by C in 54 mins. Will be covered by B in:

a) 18 min b) 37 min

c) 38 min d) 9min

LILLIPUTIAN: A TRIVIAL OR VERY SMALL, NARROW-

MINDED, PAROCHIAL, SECTARIAN

14. Walking at 5

6 of its usual speed, a train is 10 minutes too

late. Its usual time to cover the journey is:

a) 40 min b) 50 min

c) 60 min d) 45 min

15. A and B are two stations 390 km apart. A train starts from

A at 10 a.m. and travels towards B at 65 kmph. Another

train starts from B at 11 a.m. and travels towards A at

35kmph. At what time do they meet:

a) 2.20 p.m. b) 1.15 p.m.

c) 3.20 p.m d) 2.15 pm

16. A thief is spotted by a policeman from a distance of 100

metres. When the policeman starts the chase, the thief

also starts running. If the speed of the thief be 8 km/hr and

that of the policeman 10km/hr, how far the thief will have

run before he is overtaken:

a) 200m b) 300m

c) 400m d) 500m

17. PROBLEMS ON TRAIN

1. A speed of 108kmph in m/s is:

a) 10.8 b) 18

b) 30 d) 38.8

2. A speed of 14m/s in kmph is equal to:

a) 28 b) 46.6

c) 50.4 d) 70km/hr

3. In what time will a train 100 metres long cross an electric

pole, if its speed be 144km/hr:

a) 2.5 seconds b) 4.25 seconds

c) 5 seconds d) 12.5 seconds

4. How long a train 110 metres long running at the speed of

72km/hr take to cross a bridge 132 metres in length:

a) 9.8 secs b) 12.1 secs

c) 12.42 secs d) 14.3 secs

5. A train running at the speed of 60 km/hr crosses a pole in 9

seconds. What is the length of the train:

a) 120 metres b) 180 metres

c) 324 metres d) 300 metres

6. A train 132 m long passes a telegraph pole in 6 seconds. The

speed of the train is:

a) 70kmph b) 72kmph

c) 79.2kmph d) 80kmph

7. A train covers a distance of 12km in 10 minutes. If it takes 6

seconds to pass a telegraph post, then length of the train is :

a) 90m b) 100m

c) 120m d) 140m

8. A train 240m long passed a pole in 24 seconds. How long will

it take to pass a platform 650m long:

a) 65sec b) 89 sec

c) 100 sec d) 150 sec

9. The length of a bridge, which a train 130 m long and

travelling at 45 km/hr can cross in 30 seconds is:

SANGUINE: CHEERFULLY OPTIMISTIC, HOPEFUL,

BUOYANT

a) 200m b) 225m

c) 245m d) 250m

10. A train passes a station platform in 36 seconds and a man

standing on the platform in 20 seconds. If the speed of the

train is 54kmph, what is the length of the platform:

a) 120m b) 240m

c) 300m d) 360m

Page 14: JSpiders - Aptitude

www.Jspiders.com www.facebook.com/JSpiders.Basavanagudi/ Mb.9686114422 www.qspiders.com twitter.com/JBasavanagudi

11. A train takes 18 seconds to pass completely through a

station 162m long and 15 seconds through another station

120m long. The length of the train is:

a) 70m b) 80m

c) 90m d) 100m

12. A train 110 m long is running with a speed of 60kmph. In

what time will it pass a man who is running at 6kmph in

the direction opposite to that in which train is going:

a) 5 secs b) 6 secs

c) 7 secs d) 10 secs

13. Two trains 200m and 150m long are running on parallel

rails at the rate of 45kmph and 40kmph respectively. In

how much time will they cross each other, if they are

running in the same direction:

a) 72 secs b) 132 secs

c) 192 secs d) 252 secs

14. Two train 140m and 160m long run at the speed of

60kmph and 40kmph respectively in opposite direction on

parallel tracks. The time they take to cross each other is:

a) 9 sec b) 9.6secs

c) 10secs d) 10.8secs

15. Two trains are running in opposite direction with the same

speed. If the length of each train is 120 metres and they

take 12 seconds to cross each other, then speed of each

train is :

a) 10kmph b) 18kmph

c) 36 kmph d) 72kmph

16. Two trains each 100m long moving in opposite directions,

cross each other in 8 seconds. If one is moving twice as fast

as other, then speed of the faster train is :

a) 30kmph b) 45kmph

c) 60kmph d) 75kmph

17. Two trains one from Mumbai to Delhi and other from Delhi

to Mumbai start simultaneously. After they meet, the

trains reach their destination after 9 hours and 16 hours

respectively. The ratio of their speeds is:

a) 2:3 b) 4:3

c) 6:7 d) 9:16

18. BOATS AND STREAM

1. A boat can row upstream at 7kmph and downstream at

10kmph. Find the speed of boat in still water:

ANATHEMA: SOMETHING THAT ONE VEHEMENTLY

DISLIKES, ABHORRENT, HATEFUL, REPUGNANT

a) 7kmph b) 8.5kmph

c) 9kmph d) 9.5kmph

2. A man takes 3 hours 45 minutes to row a boat 15km

downstream and 2 hours 30 minutes to cover a distance of

5km upstream. Then speed of the river current is:

a) 2kmph b) 1kmph

c) 4kmph d) 6kmph

3. A man rows downstream 32 km and 14km upstream. If he

takes 6 hours to cover each distance, then velocity of the

current is:

a) 1

2 𝑘𝑚𝑝ℎ b) 1kmph

c) 1.5kmph d) 2kmph

4. If a boat goes 7km upstream in 42 minutes and speed of the

current is 3kmph, then the speed of the boat in still water

is:

a) 4.2kmph b) 9kmph

c) 13kmph d) 21kmph

5. In a stream running at 2kmph, a motorboat goes 6km

upstream and back again to the starting point in 33

minutes. Then speed of the motorboat in still water is:

ACCOLADE: honour, privilege, laurels

a) 20kmph b) 18kmph

c) 22kmph d) 24kmph

19. ALLIGATION OR MIXTURE

1. In what ratio must rice at Rs. 9.30 per kg be mixed with rice

at Rs. 10.80 per kg so that the mixture be worth Rs. 10 per

kg:

a) 7:8 b) 6:5

c) 8:7 d) 4:5

2. In what ratio must a grocer mix two verities of pulses

costing Rs. 15 and Rs 20 per kg so as to get a mixture worth

rs 16.50 per kg:

a) 3:7 b) 5:7

c) 7:3 d) 7:5

3. Find the ratio in which rice at rs. 7.20 a kg must be mixed

with a rice at 5.7 a kg to produce a mixture worth rs. 6.30 a

kg:

a) 1:3 b) 2:3

c) 3:4 d) 4:5

4. In what ratio must water be mixed with milk costing Rs. 12

per litre to obtain a mixture worth Rs. 8 per litre:

a) 1:2 b) 2:1

c) 2:3 d) 3:2

20. SIMPLE INTEREST AND COMPOUND

INTEREST

1. At the rate of 81

2% p.a. simple interest, a sum of Rs. 4800

will earn how much interest in 2 years 3 months:

MUNDANE: LACKING INTEREST OR EXCITEMENT, BORING,

TIRESOME, UNREMARKABLE

a) 796 b) 816

c) 918 d) 956

Page 15: JSpiders - Aptitude

www.Jspiders.com www.facebook.com/JSpiders.Basavanagudi/ Mb.9686114422 www.qspiders.com twitter.com/JBasavanagudi

2. The simple interest on Rs. 1820 from march 9, 2003 to may

21, 2003 at 71

2% rate will be:

a) 22.50 b) 27.30

c) 28.80 d) 29

3. How much time will it take for an amount of Rs. 450 to yield

Rs. 81 as interest at 4.5% per annum of simple interest:

a) 3.5 years b) 4 years

c) 4.5 years d) 5 years

4. A sum of Rs. 1600 gives a simple interest of 252 in 2 years

and 4 months. The rate of interest per annum is:

a) 6% b) 61

4%

c) 61

2% d) 6

3

4%

5. What will be the ratio of simple interest earned by certain

amount at the same rate of interest for 6 years and that for

9 years:

a) 1:3 b) 1:4

c) 2:3 d) 2:3

6. A sum of money at simple interest amounts to Rs. 815 in 3

years and 854 in 4 years. The sum is:

a) 650 b) 690

c) 698 d) 700

7. The simple interest on a sum of money at 8% per annum for

6 years is half the sum. The sum is:

a) 4800 b) 6000

c) 8000 d) none

8. In how much time would the simple interest on a certain

sum be 0.125 times the principal at 10% per annum:

a) 11

4 𝑦𝑒𝑎𝑟𝑠 b) 1

3

4 𝑦𝑒𝑎𝑟𝑠

c) 21

4 𝑦𝑒𝑎𝑟𝑠 d) 2

3

4 𝑦𝑒𝑎𝑟𝑠

9. At what rate percent per annum will a sum of money

double in 16 years:

a) 25% b) 61

4%

c) 26% d) 30%

10. A sum of Rs. 1550 is lent out into two parts, one at 8% and

other one at 6%. If the total annual income is Rs. 106, then

the second part is:

a) 800 b) 900

c) 1000 d) 700

11. Find compound interest on sum of 7500 at 4% per annum

for 2 years, compounded annually :

a) 610 b) 620

c) 612 d) 650

12. Find compound interest on Rs. 8000 at 15% per annum for

2 years 4 months, compounded annually :

a) 3110 b) 11109

c) 3109 d) 4000

13. Compound interest on Rs. 5000 for 11

2 𝑦𝑒𝑎𝑟𝑠 compounded

half yearly:

SOLACE: COMFORT OR CONSOLATION IN TIMES OF

DISTRESS, SUPPORT

a) 306.04 b) 307.5

c) 308 d) 309.4

14. The compound interest on Rs 30,000 at 7% per annum is Rs

4347. Then time in years is:

a) 2 b) 21

2

c) 3 d) 4

15. The principal that amounts to Rs. 4913 in 3 years at 61

4%

per annum compounded annually is :

a) 3096 b) 4076

c) 4085 d) 4096

16. The difference between compound interest and simple

interest on a amount of Rs. 15,000 for 2 years is Rs. 96.

What is the rate of interest per annum:

a) 8 b) 10

c) 12 d) none

17. The difference between simple interest and compound

interest compounded annually on a certain sum of money

for 2 years at 4% per annum is Rs. 1. The sum is :

a) 625 b) 630

c) 640 d) 650

18. The sum of money invested at compound interest amounts

to Rs. 800 in 3 years and to 840 in 4 years. The rate of

interest per annum is:

a) 21

2% b) 4%

c) 5% d) 62

3%

19. A sum of money invested at compound interest amounts to

Rs. 4624 in 2 years and to Rs. 4913 in 3 years. The sum of

money is:

a) 4096 b) 4260

c) 4335 d) 4360

20. A sum of Rs. 12000 deposited at compound interest

becomes double after 5 years. After 20 years it will become:

a) 96000 b) 1,20,000

c) 1,24,000 d) 1,92,000

21. If the simple interest on a sum of money at 5% per annum

for 3 years is Rs. 1200, Find the compound interest on the

same sum for the same period of time and at same rate:

a) 1200 b) 1250

c) 1261 d) 1300

22. In what time will Rs. 1000 become Rs. 1331 at 10% per

annum compounded annually:

a) 2 years b) 3 years

c) 4 years d) 5 years

23. The difference between the compound interest and simple

interest on an amount of Rs. 18,000 in 2 years was Rs. 405.

What was the rate of interest:

a) 10% b) 12%

c) 15% d) 18%

24. A sum of money amount to Rs. 6690 after 3 years and Rs.

10,035 after 6 years on compound interest. The sum is:

a) 4400 b) 4500

c) 4460 d) 4600

Page 16: JSpiders - Aptitude

www.Jspiders.com www.facebook.com/JSpiders.Basavanagudi/ Mb.9686114422 www.qspiders.com twitter.com/JBasavanagudi

QUAGMIRE: A DIFFICULT, COMPLICATED OR UNPLEASANT

SITUATION

25. A sum of money doubles itself at compound interest in 15

years. In how many years will it become eight times:

a) 30 years b) 40 years

c) 45 years d) 50 years

21. AREA, SURFACE AREAS AND VOLUMES

1. Find the cost of fencing a rectangular plot of length 30m

and breadth 20m at the rate of 5 Rs. Per metre:

a) 400m b) 500m

c) 600m d) 700m

2. One side of a rectangular field is 15m and one of its

diagonals is 17m. then the area of the field(in 𝑚2) is:

a) 130 b) 110

c) 120 d) 140

3. If the diagonal of a rectangle is 17 cm long and its perimeter

is 46cm, find the area of the rectangle:

a) 110𝑐𝑚2 b) 120𝑐𝑚2

c) 140𝑐𝑚2 d) 150 𝑐𝑚2

4. If the length and breadth of a rectangular plot be increased

by 50% and 20% respectively, then how many times will its

area be increased:

a) 11

3 b) 2

c) 32

5 d) 4

1

5

5. The percentage increase in the area of a rectangle, if each

of its side is increased by 20% is :

a) 40% b) 42%

c) 44% d) 46%

6. A towel when bleached was found to have lost 20% of its

length and 10% of its breadth. The percentage of decrease

in area is:

a) 10% b) 10.08%

c) 20% d) 28%

7. The perimeter of two squares are 40cm and 32cm. find the

perimeter of the third square whose area is equal to the

difference of the areas of the two squares:

a) 20cm b) 24cm

c) 22cm d) 18cm

8. A room 5m 55cm long and 3m 74cm broad is to be paved

with squares tiles. If the side of the square is 34 cm, then

how many squares tiles needed:

a) 170 b) 176

c) 180 d) 185

9. Find the area of the square, one of whose diagonal is 3.8 m

long:

a) 7.22𝑚2 b) 8𝑚2

c) 9𝑚2 d) 10𝑚2

10. The diagonals of two squares are in the ratio 2:5, then ratio

of their areas is:

a) 4:20 b) 4:25

c) 5:32 d) 5: 27

11. If each side of a square is increased by 25%, then % change

in its area is:

RAMIFICATION: CONSEQUENCE, RESULT, AFTERMATH,

IMPLICATION

a) 55.25% b) 56.75%

c) 56.25% d) 54.55%

12. The area of a triangle , whose sides measure 13cm, 14cm

and 15cm:

a) 80% b) 81%

c) 84% d) 85%

13. Find the length of the altitude of an equilateral triangle of

side 3√3 𝑐𝑚:

a) 4cm b) 4.5cm

c) 5cm d) 5.5cm

14. In two triangles, the ratio of the areas is 4:3 and ratio of

their heights is 3:4. Then ratio of their bases is :

a) 9:16 b) 4:5

c) 16:9 d) 7:9

15. The base of parallelogram is twice its height. If the area of

the parallelogram is 72sq. cm, then its height is :

a) 6cm b) 12cm

c) 15cm d) 8cm

16. Find the area of a rhombus one side of which measure

20cm and one diagonal is 24cm:

a) 380 sq.cm b) 384 sq.cm

c) 390 sq.cm d) 400 sq.cm

17. The difference of two parallel sides of a trapezium is 2cm.

the perpendicular distance between them is 19cm. if the

area of trapezium is 475 sq.cm , then length of the parallel

sides is:

a) 26cm, 24cm b) 28cm, 26cm

c) 32cm, 30cm d) 42cm, 40cm

18. A wheel makes 1000 revolutions in covering a distance of

88km. find the radius of the wheel:

a) 12m b) 10m

c) 11m d) 14m

19. A sector of 120°, cut out from a circle, has an area of

93

7𝑠𝑞. 𝑐𝑚 , then radius of the circle is :

a) 1cm b) 2cm

c) 3cm d) 4cm

20. The ratio of the areas of the incircle and circumcircle of a

square:

a) 1:2 b) 2:5

c) 3:4 d) 2:3

21. If the radius of a circle is decreased by 50%, then %

decrease in its area is:

a) 50% b) 60%

c) 70% d) 75%

22. If the length of the diagonal of a square is 20cm, then its

perimeter must be:

a) 10√2 𝑐𝑚 b) 40cm

c) 40√2 𝑐𝑚 d) 200cm

Page 17: JSpiders - Aptitude

www.Jspiders.com www.facebook.com/JSpiders.Basavanagudi/ Mb.9686114422 www.qspiders.com twitter.com/JBasavanagudi

23. A square and a rectangle have equal areas. If their

perimeter are p1 and p2 respectively, then:

a) P1<p2 b) p1=p2

c) P1>p2 d) none

UNBRIDLED: UNCONTROLLED, UNRESTRICTED,

UNSTOPPABLE

24. If the perimeter of a square and a rectangle are same, then

the area A and B enclosed by them would satisfy the

condition:

a) A<B b) A≤ 𝐵

c) A>B d) A≥ 𝐵

25. The ratio of the areas of two squares, one having its

diagonal double than the other is:

a) 2:1 b) 2:3

c) 3:1 d) 4:1

26. The ratio of the area of a square to that of the square on its

diagonal is:

a) 1:2 b) 2:3

c) 3:4 d) 4:5

27. If the area of an equilateral triangle is 24√3 𝑐𝑚 , then its

perimeter is:

a) 2√6 𝑐𝑚 b) 4√6 𝑐𝑚

c) 12√6 𝑐𝑚 d) 96cm

28. The diameter of a wheel is 1.26m. how far will it travel in

500 revolution:

a) 1492m b) 1980m

c) 2530m d) 2880m

29. The number of revolution a wheel of diameter 40cm makes

in travelling a distance of 176m, is:

a) 140 b) 150

c) 160 d) 166

30. A wire can be bent in the form of a circle of radius 56cm. if

it is bent in the form of an square, then its area will be:

a) 3520 sq.cm b) 6400 sq. cm

c) 7744 sq.cm d) 8800sq.cm

31. If the ratio of areas of two circles is 4:9, then the ratio of

their circumference will be:

a) 2:3 b) 3:2

c) 4:9 d) 9:4

32. The area of the incircle of an equilateral triangle of side

42cm is:

a) 22√3 𝑠𝑞. 𝑐𝑚 b) 231 sq.cm

c) 462 sq.cm d) 924 sq.cm

33. Length of a longest pole that can be placed in a room 12m

long, 8m broad and 9m high is:

a) 15m b) 16m

c) 17m d) 18m

34. The diagonal of a cube is 6√3𝑚. Its volume is:

a) 216 cubic m b) 217 cubic m

c) 220 cubic m d) 222 cubic m

35. If the edge of a cube is increased by 50%, then % increase in

its surface area is:

a) 150% b) 140%

c) 125% d) 160%

36. Two cubes have their volumes in the ratio 1:27. Then ratio

of their surface area is:

a) 1:3 b) 1:9

c) 1:27 d) 1:2

CHIMERICAL: ILLUSION, FANTASY, DELUSION, FANCY

37. If the capacity of a cylindrical tank is 1848 cubic m ,and

diameter of its base is 14m, then depth of the tank is:

a) 10m b) 12m

c) 14m d) 15m

38. The radii of two cylinders are in the ratio 3:5, and their

heights are in the ratio 2:3. Then ratio of curved surface

areas is:

a) 2:3 b) 2:5

c) 3:5 d) 3:4

39. The radii of the base of a cylinder and a cone are in the

ratio 3:4 and their heights are in ratio 2:3. Then ratio of

their volumes is:

a) 8:9 b) 9:8

c) 8:5 d) 5:8

40. If the radius of a sphere is increased by 50%, find the

increase % in its surface area:

a) 120% b) 125%

c) 130% d) 140%

41. Find the number of lead balls, each 1cm in diameter that

can be made from a sphere diameter 12 cm:

a) 1726 b) 1730

c) 1736 d) 1728

22. CLOCKS

1. A clock is started at noon. By 10 minutes past 5, the hour

hand has turned through:

a) 145° b) 150°

c) 155° d) 160°

2. At 3.40, the hour hand and the minute hand form an angle

of :

a) 120° b) 125°

c) 130° d) 135°

3. The angle between minute hand and hour hand of a clock

when the time is 4:20 is:

a) 0° b) 10°

c) 5° d) 20°

4. At what angle the hands of a clock are inclined at 15

minutes past 5:

a) 581°

2 b) 64

c) 671°

2 d) 72

2

5. At what time between 2 and 3 o’clock will the hands of a

clock be together:

a) 10 mins past 2 b) 1010

11𝑚𝑖𝑛 pst 2

c) 5 mins past 2 d) 20 mins past 2

6. How many times do the hands of a clock coincide in a day:

Page 18: JSpiders - Aptitude

www.Jspiders.com www.facebook.com/JSpiders.Basavanagudi/ Mb.9686114422 www.qspiders.com twitter.com/JBasavanagudi

a) 20 b) 21

c) 22 d) 24

7. How many times in a day the hands of a clock are straight:

a) 22 b) 24

c) 44 d) 48

MISOLOGIST: ONE WHO HATES OR DISLIKES REASONING

OR ARGUMENT

8. How many times are the hands of a clock are at right angle

in a day;

a) 22 b) 24

c) 44 d) 48

9. How many times in a day clock of the hands are in straight

line but opposite in direction:

a) 20 b) 22

c) 24 d) 48

23. PERMUTATION AND COMBINATION

1. 𝟑𝟎!

𝟐𝟖! =

a) 860 b) 870

c) 880 d) 890

2. 25P2=

a) 500 b) 600

b) 700 d) 750

3. 10C3=

a) 110 b) 120

c) 130 d) 140

4. In how many ways can letters of the word ‘ORANGE’ can be

arranged:

a) 120 b) 130

c) 720 d) 740

5. How many words can be formed using the word

‘DAUGHTER’ so that all vowels always come together:

a) 4000 b) 4320

c) 5000 d) 5320

6. How many words can be formed so that all vowels never

come together:

a) 100 b) 120

c) 130 d) 140

7. How many words can me formed using the letter

“DIRECTOR” so that vowels are always together:

a) 2100 b) 2160

c) 2200 d) 2400

8. In how many ways can cricket eleven be chosen out of a

batch of a 15 players:

a) 1300 b) 1360

c) 1365 d) 1400

9. In how many ways a committee of 5 members can be

selected from 6 men and 5 ladies, consisting of 3 men and 2

ladies:

a) 210 b) 240

c) 200 d) 260

10. In how many ways can the letters of the word ‘APPLE’ be

arranged:

a) 720 b) 120

c) 60 d) 180

11. In how many ways can the letters of the word ‘LEADER’ can

be arranged:

a) 72 b) 144

c) 360 d) 720

DESPISE: HATE, LOATHE, DEPLORE, ABOMINATE

12. In how many ways can the word ‘JUDGE’ be arranged so

that the vowels always come together:

a) 48 b) 120

c) 124 d) 160

13. In how many ways can a group of 5 men and 2 women be

made out of total 7 men and 3 women:

a) 63 b) 90

c) 126 d) 45

14. A box contains 2 white balls, 3 black balls and 4 red balls. In

how many ways can 3 balls be drawn from the box, if at

least one black ball is to be included in the draw:

a) 32 b) 48

c) 64 d) 96

24. PROBABILITY

1. In a throw of a coin, find the probability of getting a head:

a) 1

3 b)

1

2 c)

1

4 d)

1

5

2. In a simultaneous throw of two coins, the probability of

getting at least one head is:

a) 2

3 b)

1

2 c)

1

3 d)

3

4

3. Three unbiased coins are tossed. What is the probability of

getting at least two heads:

a) 1

4 b)

1

3 c)

1

2 d)

1

8

4. Three unbiased coins are tossed. What is the probability of

getting at most two heads:

a) 3

4 b)

1

4 c)

3

8 d)

7

8

5. In a single throw of dice what is the probability of getting a

number greater then 4:

a) 1

3 b)

1

2 c)

2

3 d)

1

4

6. What is the probability of getting a sum of 7 from two

throw of a dice:

a) 1

6 b)

1

4 c)

2

3 d)

3

4

7. In a simultaneous throw of two dice, what is the probability

of getting a doublet:

a) 1

6 b)

1

4 c)

2

3 d)

3

7

8. in a simultaneous throw of a pair of dice, find the probability

of getting a total more than 7:

a) 7

12 b)

5

12 c)

5

6 d)

1

3

Page 19: JSpiders - Aptitude

www.Jspiders.com www.facebook.com/JSpiders.Basavanagudi/ Mb.9686114422 www.qspiders.com twitter.com/JBasavanagudi

9. two dice are thrown together. What is the probability that

the sum of the numbers on the two faces is divisible by 4 or

6:

a) 5

18 b)

4

18 c)

7

18 d)

11

18

10. one card is drawn from a pack of 52 cards. What is the

probability that the card drawn is face card:

a) 1

13 b)

4

13 c)

1

4 d)

9

52

11. the probability that a card drawn from a pack of 52 cards

will be a diamond or a king is:

a) 2

13 b)

4

13 c)

1

13 d)

1

52

CHURLISH: RUDE AND IMPOLITE, CYNICAL, WASPISH,

GRUMPY, TRUCULENT

25. HEIGHTS AND DISTANCE

1. If the height of a pole is 2√3𝑚 and the length of its shadow

is 2m, then the elevation of the sun is:

a) 45° b) 60° c) 30° d) 90°

2. A ladder leaning against the wall makes an angle of 60° with

the ground. If the length of the ladder is 19m, then distance

of the ladder from the wall is:

a) 8m b) 9m c) 8.5m d) 10m

3. The angle of the elevation of the top of a tower at a point

on the ground is 30°. On walking 24m towards the tower,

the angle of elevation becomes 60°. Then height of the

tower is:

a) 20m b) 20.5m c) 20.76m d) 20.85m

4. A man standing on the bank of a river observes that the

angle subtended by a tree on the opposite side bank is 60°.

When he retires 36m from the river bank ,he finds the angle

to be 30°, then the width of the river is:

a) 15m b) 16m c) 17m d) 18m

5. The top of a 15m high tower makes an angle of elevation of

60° with the bottom of an electric pole and angle of

elevation of 30° with the pole. Then the height of the

electric pole is:

a) 5m b) 8m c) 10m d) 12m

26. ODD MAN OUT AND SERIES

Direction : insert the missing number

1. 6,12,21,?,48

a) 33 b) 38 c) 40 d) 45

2. 120, 99,80,63,48,?

a) 35 b) 38 c) 39 d)40

3. 2, 9, 28,65, 126, ?

a) 200 b) 212 c) 217 d) 220

4. 0, 7, 26, 63, 124, ?

a) 210 b) 215 c) 214 d) 220

5. 0, 6, 24, 60, 120 ,?

a) 200 b) 205 c) 210 d) 215

6. 3, 5, 7, 11, 13, ?

a) 15 b) 16 c) 17 d) 19

7. 9, 25, 49, 121, 169, ?

a) 289 b) 280 c) 240 d) 220

Find the odd man out:

8. 3, 5, 7, 12, 17, 19

a) 19 b) 17 c) 13 d) 12

9. 10, 14, 16, 18, 21, 24, 26

a) 26 b) 24 c) 21 d) 18

10. 6, 9, 15, 21, 24, 28, 30

a) 28 b) 21 c) 24 d) 30

11. 1,5, 14, 30, 50, 55, 91

a) 5 b) 50

c) 55 d)91

BESTIAL: CRUEL AND INHUMANE

12. 2, 5, 10, 17,26, 37, 50, 64

a) 50 b) 26

c) 37 d) 64

13. 19, 28, 39,52, 67, 84, 102

a) 52 b) 102

c) 84 d) 67

14. 4, 5, 7, 10, 14, 18, 25, 32

a) 7 b) 14

c) 18 d) 32

15. 1, 2 ,6, 15, 31, 56, 91

a) 31 b) 91 c) 56 d)15

27. GEOMETRY

1. Sum of total angles of a pentagon is:

a) 500° b)540°

c) 600° d) 720°

2. One angle of a regular hexagon is equal to:

a) 110° b) 120°

c) 150° d) 60°

3. Sum of all exterior angles of a polygon is equal to:

a) 180° b) 300° c) 360° d) 400°

4. One exterior angle of a regular hexagon is equal to:

a) 50° b) 120° c) 60° d) 90°

28. RACES

1. In a km race A beats B by 28 metres or 7 seconds. Then

A’s time over the course is:

a) 4 min b) 4 min 10 secs

c) 4 min 3 secs d) 5 min.

2. A can run 1 km in 3 min 10 secs and B can over

the same distance in 3 min 20 secs. By what

distance can A beat B:

a) 40m b) 60m

c) 80m d) 50m

Page 20: JSpiders - Aptitude

www.Jspiders.com www.facebook.com/JSpiders.Basavanagudi/ Mb.9686114422 www.qspiders.com twitter.com/JBasavanagudi

3. In a 100m race, A covers the distance in 36

seconds and B in 45 seconds. In this race A beats

B by:

a) 10m b) 20m

c) 30m d) 40m

4. In a 200m race A beats B by 35m or 7 seconds. A’s

time over the course is;

a) 40sec b) 47secs

c) 33 secs d) none

5. A can run 22.5m while B can run 25m. in a km

race B beats A by:

a) 50m b) 100m

c) 60m d) 80m

29. CALENDER

1. January 1, 2007 was Monday. What day of the week

lies on jan. 1,2008:

IMMACULATE: PERFECT FROM ALL ANGLES, WITHOUT

ANY MISTAKES

a) Monday b) Tuesday

c) Wednesday d) Sunday

2. January 1, 2008 was Tuesday. What day of the week lies

on January 1, 2009:

a) Monday b) Wednesday

c) Thursday d) subday

3. On8th dec. 2007 Saturday falls. What day of the week

was on 8th dec. 2006:

a) Sunday b) Thursday

c) Tuesday d) Friday

4. What day of the week will fall on jan 10, 2017:

a) Sunday b) Monday

b) Tuesday d) Wednesday

30. SERIES

1. Find 35th term of the series 3,7,11,15….

2. Find 100th term of 8, 11, 15, 19,…….

3. Find sum of 20 terms of A.P 5, 8, 11,14, 17 …..

4. Find sum of first 30 terms of 1,3,5,7,9 …..

5. Find sum of first 25 terms of A.P 2,4,6,8,10, …..

6. Find the sum of 40 terms of 7,10,13,16,….

7. Fin 8th term of 3,6,12,24, …..

8. Find 7th term of 4,8,16, 32…..

9. Find sum of first 8 terns of G.P 2,4,8,16….

31. SET THEORY

1. In a class 40% of students enrolled for mathematics and

70% enrolled for economics. If 15% enrolled for both the

subject, then how many % of students did not enrol for

either of the two subjects:

a) 5% b) 10% c) 15% d) 20%

2. In a group of 50 musicians, 28 play pop music and 20 play

classical music. If 10 musicians play neither of the two

kinds of music, then how many musicians play both kind

of music:

a) 10 b) 12 c) 8 d) 14

3. In a group of 40 tourists, 12 knew both English and

German and 22 knew German. If the tourist knew at least

one of the two languages , then how many tourist knew

only English and not german:

a) 28 b) 10 c) 18 d) 32

4. In a school, 42% students like cricket, 5% like cricket and

football and 10% like none. If 96 students like only

football, then how many students like only cricket:

a) 53 b) 37 c) 74 d) 36 e) 30

5. In an organization 30% of the employee are matriculates,

50% of the remaining is graduates and remaining 180 are

post-graduates. how many employee are graduates:

a) 360 b) 240

c) 300 d) 180

DENIZEN: INHABITANT, RESIDENT, NATIVE, LOCAL

32. LINEAR EQUATIONS

1. Solve:

X+y+z=6,

2x+y+z=7

X+2y+3z=14

2. X+2y+z=7

X+2z=8

X+2y=4

3. X+2y=7

Y+2z=7

Z+2x=4

33. QUADRATIC EQUATION

1. Solve: 3𝑥2 + 7𝑥 + 2 = 0

2. Solve: 4𝑥2 − 9𝑥 + 2 = 0

3. Solve: 6𝑥2 − 14𝑥 + 4 = 0

4. Solve: 8𝑥2 + 18𝑥 + 4 = 0

5. Solve: 3𝑥2 − 2√6𝑥 + 2 = 0

6. Solve: √2𝑥2 + 7𝑥 + 5√2 = 0

BEQUEATH: leave property to a person by will,

donate, bestow on, endow with

GENERAL ENGLISH

1. TENSE

Tense denotes the time of action as well as the state.

Types of tense:

1. Present tense

2. Past tense

3. Future tense

PRESENT TENSE:

Page 21: JSpiders - Aptitude

www.Jspiders.com www.facebook.com/JSpiders.Basavanagudi/ Mb.9686114422 www.qspiders.com twitter.com/JBasavanagudi

1. Present Indefinite Tense: following activities are

included under present indefinite tense:

Regular action:- I come here daily.

Irregular activities:- earthquake comes in

Japan.

Habits:- he smokes.

Universal truth:- the sun rises in the east.

In newspaper headlines and commentary of

sports:- PM signs deal.

To express planned action of near future.

PM leaves for china next week.

Form: sub+ v1+obj

2. Present Continuous Tense: An action that is going on at

the time of speaking comes in present continuous tense.

I am studying English now.

Coastal areas are getting submerged.

For events that will take place in near

future.

I am going to Mumbai tomorrow.

Form: sub+ is/am/are+ v1+ing+obj

3. Present perfect tense: an action that has recently

finished comes under present perfect tense.

He has come to Delhi recently.

If the action is important and not the time of

action, present perfect tense is used.

Science has given us many new inventions.

We have reached the moon.

Usually, recently, already, yet, so far are used in

perfect tense.

He has not reached home yet.

I haven’t seen him since he left India.

Since he joined army he has not taken any leave.

Form: sub+ has/have+v3+obj

4. Present perfect continuous tense: An action already

started and still going on comes under present perfect

continuous tense.

I have been living in Delhi for five years.

I have been teaching you for an hour.

FOR is used for a period of time.

E.g.: for two hours, for last 2 years

SINCE: is used for a point of time.

Since Monday,since 2008, since 7pm.

FORM: sub+has/have+been+v1+ing+obj+

for/since +time

PAST TENSE:

1. SIMPLE PAST TENSE: an action that is completed is

called simple past.

I saw you but you didn’t see me.

He did make a mistake.

If the sentence starts with ‘it is time’ or ‘it’s high time’

the verb should be in v2 form.

It’s time you studied.

It is high time you realised.

If past time is given in a sentence, the sentence should

be in simple past.

I came here yesterday.

FORM: sub+v2+obj

Sub+didn’t+v1+obj

2. PAST CONTINUOUS TENSE: if an action was in

continuous in the past, simple past tense is used.

I was waiting for u.

I was going to meet him.

In imaginary sentences were is used no matter it is

used as a helping verb or main verb.

I wish, I were a bird.

He pretended as if he were sleeping.

FORM: sub+was/were+v1+ing+obj

3. PAST PERFECT TENSE: it is used to express an action

which has occurred in past (usually, a long time ago)

and action which has occurred in past before another

action in past.

I had seen him before he stopped his car.

Before he understood anything the robber had fled.

By the time I reached the theatre the show had started.

If two actions take place in the past, one after the

other, the first action will be in past perfect tense and

the second action will be in simple past tense.

FORM: sub+had+v3+obj

4. PAST PERFECT CONTINUOUS TENSE: an activity that

started in the past, continued and finished in past

comes under past perfect continuous tense.

I had been waiting for you since morning.

They had been playing for ten years.

She had been dancing since 1980.

Future tense:

1. SIMPLE FUTURE: an action to take place in future

comes under simple future tense.

I shall meet you tomorrow.

FORM: sub+shall/will+v1+obj

2. FUTURE CONTINUOUS TENSE: an action going on

in future comes under future continuous tense.

We shall be taking the examination at this time, next

month.

Some verbs do not take ‘ing’ form. Hence they

cannot come in continuous tense.

e.g. see, taste, smell,hear,prefer, please, notice,

recognise, think, know, mean, mind, remember,

own, have, belong, comprise, belive, like, dislike,

love, adore, want, wish, desire, hate, agree, trust,

imagine, look, seem, appear, hope, refuse.

NOTE:a) if have is used to denote have fun or to eat,

have can be used in ‘ing’ form.

b) in ‘gerund’ and ‘present participle’ verbs are used

in ‘ing’ form.

Being ill I could not come.

Seeing is believing.

If verb is used after a preposition, the verb will be in

‘ing’ form.

Bats are capable of hearing ultrasonic waves.

We must keep away from smoking.

FORM: sub+shall/will+be+v1+ing+obj

3. FUTURE PERFECT TENSE: an action that will have

been completed in future , comes under future

perfect tense.

You will have finished your syllabus by the end of

next month.

By the time I reach the station, the train will have

left.

FORM: sub+shall/will+have+v3+obj

Page 22: JSpiders - Aptitude

www.Jspiders.com www.facebook.com/JSpiders.Basavanagudi/ Mb.9686114422 www.qspiders.com twitter.com/JBasavanagudi

4. Future perfect continuous tense: an action that

continues for some time comes under future

perfect continuous tense.

I shall have been living in Bangalore for five years by

the end of this year

FORM: sub+shall/will+have+been+v1+ing+

Obj+for/from+time

SPOTTING THE ERROR:

1. a) I have not seen him since twenty years/b) and so I

can not say with certainty/ c) whether he is in India

or not./d) no error

2. a) Although I have been playing cricket/b) for more

than three years/c) I have not been able to score a

century/d) no error

3. a) By the time/b) we got our tickets and entered the

cinema theatre,/ c) the show was already begun./d)

no error

4. a) I have passed/b) the examination/c) two years

ago/d) no error

5. a)This TV serial/b) is going on/c) for three years/d)

no error

6. a) As soon as the CEO entered the office, the/b)union

leaders approached him/c) and reported the matter

to him./d) no error.

7. a) My uncle/ b) has left/c) for Bombay last

Saturday./d) no error.

8. a) Good heavens!/b) how has she /c) grown!/d) no

error

9. a) I ate/b) nothing/c) since morning/d) no error.

10. a) Madhuri Dixit /b) is having /c) a large fan

following./d) no error

11. a) It is time/b) we should have done/c) something

useful/d) no error

12. a) I am going to buy/ b) a computer /c) when the

prices comes down./d) no error.

13. a) I wish/b) I have learnt swimming/c) when I was

young./d) no error.

2. SUBJECT VERB AGREEMENT

In English, the verb must agree with its subject in number and person. In other words, the verb must be of the same number and person as the subject. Subjects and verbs must agree in number. This is the main rule that forms the background of the concept.

RULE 1

Subjects don’t always come before verbs in questions. Make sure you accurately identify the subject before deciding on the proper verb form to use.

Eg Where are the pieces of this puzzle? (The subject is not this puzzle but it is the pieces of this puzzle so the subject is plural)

The study of languages and sciences is very important for your study. (The subject is not languages and sciences but it is study.)

RULE 2

When two singular nouns are connected by and the verb used will be plural.

Eg The boy and the girl are waiting outside the school.

Important note

A) But when the two nouns connected by and together represent the same person, same thing or same idea the verb used will be singular.

Eg The principal and science teacher has not come to the school today. (Here the principal is also the science teacher)

B) If two different singular nouns express one idea, the verb should be in the singular form. Or the two singular nouns/uncountable nouns are used in pairs then the verb used will be singular.

The law and order situation in the state are under control ✘

The law and order situation in the state is under control ✔

Profit and loss is a part of business. (Profit and loss goes in pairs so a singular verb (is)

Hard work and luck takes one to success.

RULE 3

‘Unlike’, no less than’, ‘nothing but’ ‘As well as’, ‘with’, ‘alongwith’, ‘together with’, ‘and not’, ‘In addition to’, ‘but’, ‘besides’, ‘except’, ‘rather than’, ‘accompained by’, ‘like’ If these words are the connecting words that join

two subjects then the verb used will be according to the first

subject.

(1) My sister unlike my brothers wishes to have a career in engineering.

(2) The actress, along with her manager and some

friends, are invited to the function ✘

The actress, along with her manager and some

friends, is invited to the function ✔

RULE 4

When two or more Subjects are connected by or, nor, either … or, neither … nor, the Verb is according to the subject that falls close to the verb.

Examples:

1 Either James or John are to be promoted (incorrect)

Either James or John is to be promoted (correct)

2 Neither the girls nor he are to take up this task (incorrect)

Neither you nor he is to take up this task (correct)

3 Either the Chief Minister or the Cabinet Ministers is responsible for this problem. (incorrect)

Page 23: JSpiders - Aptitude

www.Jspiders.com www.facebook.com/JSpiders.Basavanagudi/ Mb.9686114422 www.qspiders.com twitter.com/JBasavanagudi

Either the Chief Minister or the Cabinet Ministers are responsible for this problem. (Correct)

4 Either you or I are responsible for this mistake. (Incorrect)

Either you or I am responsible for this mistake. (Correct)

RULE 5

Usage of either,neither,none etc.

‘Neither of……’ It is only used in case of two things or

persons and not one of the two.

Neither of his five sons settled in India. (incorrect)

None of his four sons looked after him. (correct)

‘Either of……”It means the choice is between two

persons and things and one of the two.

Either of the three children has broken the window. (incorrect)

One of……….. It is used to select one out of something

that has more than two.

One of the six boys admitted before the principal that they had all cheated in the exam.(correct)

‘Both’/‘not’

We do not use both and not in the same sentence. If we

wish to state that out of the two thing not even one of them

then use neither of.

Both of the girls did not reach the examination centre on

time. (incorrect)

Neither of the girls reached the examination centre on time.

(correct)

RULE 6

When ‘not only ……. but also’ is used to combine two subject, the verb agrees with the subject close to it.

Examples:

Not only silver, but also gold are mined in this country ✘

Not only silver, but also gold is mined in this country ✔

RULE 7

When two nouns joined by and have their own articles then we consider them as plural and plural verb is used.

Examples:

The director and the producer is shooting in Himachal. (✘)

The director and the producer are shooting in Himachal. (✔)

But when two nouns joined by and do not have their own articles then we consider them as singular and singular verb is used. The article is used only before the first noun.

Examples:

The principal and director of the school is on leave for three days.

A large and comfortable chair is needed in my study.

A black and a white cat —– means two cats.

A black and white cat ——–only one cat with both colours black and white.

RULE 8

Majority can be singular or plural. If it is alone it is usually singular, if it is followed by a plural noun, it is usually plural.

The majority believe that the country can progress ✘

The majority believes that the country can progress ✔

The majority of the lecturers believes that the student has

not copied in the examination ✘

The majority of the lecturers believe that the student has not

copied in the examination ✔

RULE 9

Collective nouns

Many words indicating a number of people or animals are singular. The following nouns are usually singular. Examples of collective nouns

Family, group, committee, class, organisation, team, army, club, crowd, government, jury, minority, public

Examples :

The committee have met and accepted the proposal ✘

The committee has met and accepted the proposal ✔

The family were happy at the news ✘

The family was happy at the news ✔

The crowd was wild with excitement ✔

Congress has initiated a new plan to combat inflation ✔

Page 24: JSpiders - Aptitude

www.Jspiders.com www.facebook.com/JSpiders.Basavanagudi/ Mb.9686114422 www.qspiders.com twitter.com/JBasavanagudi

Our team is certain to win the match ✔

The family living next door often quarrel among themselves

Note:

A collective noun is treated as plural when the group it names is considered to be made up of individuals. Because members of the group can act on their own, the word is considered plural.

The play’s cast are rehearsing their lines.

The plural subject cast requires the plural verb are because the members of the cast are functioning as individual people doing separate things.

The jury often have different reactions to the evidence they hear.

The plural subject jury requires the plural verb have because the members of the jury are being considered as individuals.

RULE 10

Generally with a plural number we use a plural verb.

But or nouns indicating a specific time, money, and measurements (weight , distance and height) used as a whole are singular and take a singular verb.

Examples:

Twenty-five rupees are not such big amount for him ✘

Twenty-five rupees is not such big amount for him ✔

Two miles are too much for this man to run ✘

Two miles is too much for this man to run ✔

But when they are not used as a whole and are further subdivided into smaller units we use a plural verb.

www.ssccglpinnacle.com

Note the difference-

Twenty thousand a handsome salary. (Treated as a singular

unit)

Twenty thousand been spent on different useful

commodities such as rent , grocery, fees etc.

RULE 11

When a lot of , a great deal of, plenty of, most of, and some of refer to number (countable noun) , a plural verb is used.

Examples:

A lot of people was present in the gallery some of the

students were absent ✘

A lot of people were present in the gallery some of the

students were absent ✔

Note : If these expressions refer to an amount (Uncountable noun) , the verb is in the singular number.

A lot of work has to be completed before we go ✔

A great deal of work has been finished ✔

RULE 12

Names of countries are always singular. Certain names like West indies and United States may seem to be plural as they end in s but they are singular. But in sports, while referring to the players, the name of the country is followed by plural verb. Examples:

England has won the World Cup ✘

England have won the World Cup. ✔

RULE 13

When the ‘enemy’ is used in the sense “armed forces” of a nation with which one’s country is at war, we have to use the plural verb. www.ssccglpinnacle.com

Example :The enemy were forced to retreat.

RULE 14

Structure: none + of the + non-count noun + singular verb

Examples:

None of the counterfeit money have been found ✘

None of the counterfeit money has been found ✔

Structure: none + of the + plural count noun + plural verb

Examples:

None of the students has finished the exam yet ✘

None of the students have finished the exam yet ✔

No can take either a singular or plural verb depending on the noun which follows it.

Structure: No + singular noun + singular verb

No example is relevant to this case ✔

Structure: No + plural noun + plural verb

No examples are relevant to this case ✔

Page 25: JSpiders - Aptitude

www.Jspiders.com www.facebook.com/JSpiders.Basavanagudi/ Mb.9686114422 www.qspiders.com twitter.com/JBasavanagudi

RULE 15

A number of / the number

Observe the two structures:

(i) a number of + plural noun + plural verb.

(ii) the number of + plural noun + singular verb.

Examples:

A number of students is going to the class picnic ✘

A number of students are going to the class picnic ✔

The number of days in a week are seven ✘

The number of days in a week is seven ✔

The number of residents who have been residing in this

colony is quite small ✔

A number of the applicants have already been interviewed

RULE 16

Relative pronouns

Sometimes the pronoun who, that, or which is the subject of a verb in the middle of the sentence. The pronouns who, that, and which become singular or plural according to the noun directly in front of them. So, if that noun is singular, use a singular verb. If it is plural, use a plural verb.”

The verb should not be chosen according to the subject of the sentence.

Example:

“He is one of the men who do the work.

The word in front of who is men, which is plural. Therefore, use the plural verb do.”

RULE 17

The word were replaces was in sentences that express a wish or are contrary to fact:

Example: If the teacher were here, you’d have to pay.

The teacher is singular so it should be followed by was. But the teacher isn’t actually here, so we say were, not was. The sentence shows things that are hypothetical, wishful, imaginary, or factually contradictory. In such sentences we use were.

Examples:

I wish it were Friday.

RULE 18

The following are considered as singular.

Each, Every, Everyone, Someone, Somebody, Nobody, None, One, Any, Many a, More than one are treated as singular. We use a singular verb, singular pronoun and singular noun with them.

Examples

(1) Each students from Pinnacle has cleared the exam.

(2) Each boy and each girl has to finish his work by the weekend..

(3) More than one present there.

(4) Each man and each woman was garlanded by the host.

The word many can be used in the following ways.

1 Many a students is standing in the canteen .

Although the word preceding the verb ‘is’ is plural in nature, the correct verb is singular one ‘is’, because of the presence of ‘many a’. www.ssccglpinnacle.com

Many a will be followed by the singular noun and a singular verb will be used.

2 Many students were standing in the canteen.

3 A great many students were standing in the canteen.

Note : But if each, every, one is followed by of then the word of will be followed by a plural noun/pronoun. But the verb and pronoun used in the sentence will be singular.

One of the girls/them has made a card for her teacher.

Indefinite Pronoun –

If ‘One’ is the subject of the sentence then the pronouns used will be one’s/oneself etc. Do not use him/himself

(1) One should complete his work in time. (Incorrect)

(2) One should keep one’s promise. (Correct)

RULE 19

In optative sentences the verb used is always plural irrespective of the subject. Even with singular subjects we use a plural verb.

Examples.

1) God save the world!

2) Long live our prime minister!

3) May he achieve success.

Page 26: JSpiders - Aptitude

www.Jspiders.com www.facebook.com/JSpiders.Basavanagudi/ Mb.9686114422 www.qspiders.com twitter.com/JBasavanagudi

RULE 20

Amount of/ quantity of + Uncountable Noun the verb used will be singular.

(1) The amount of sugar are not sufficient to

prepare a cup of tea. (Incorrect)

(2) The amount of money is not sufficient (Correct)

RULE 17

All can be treated as both singular and plural.

When it refers to persons or things it is treated as plural or else it is treated as singular.

RULE 18

Furniture, advice, work, evidence, equipment, news,

information, luggage, baggage, percentage, poetry,

knowledge, dirt, dust, traffic, electricity, music,

breakage, stationary, scenery, confectionery, pottery,

bakery, crockery, behaviour areuncountable

So we use a singular verb with them.

(1) The scenery of Kashmir enchanted us.

(2) I passed but the percentage of marks is not good.

RULE 19

Headphones , knickers , premises (buildings) , Alms , ruins, amends , archives , arrears, auspices, congratulations, embers , fireworks, lodgings, outskirts, particulars, proceeds, regards, riches, remains, savings, shambles, surroundings, tidings, furnishings, earnings, leftovers, troops, tactics, thanks, valuables, forceps, wages, belongings, braces, Scissors, tongs ,pliers, pincers, bellows trousers, pants, pajamas, shorts , gallows , fangs spectacles, goggles, binoculars , eyeglasses .

These nouns are always plural and a plural verb is used with them.

Examples

These scissors are for cutting paper.

Your clothes are dirty.

Have you seen my glasses? I want to read the newspaper.

What kind of goods does your company produce?

The table of contents should not contain any pictures.

I ran up the stairs and tore the door open.

The police used firearms to disperse the crowd.

I live on the outskirts of the city.

Some of these nouns are often used with the expression a pair of, as they refer to things made up of two parts:

a pair of trousers, a pair of jeans, a pair of shoes

a pair of slippers, a pair of glasses, a pair of gloves

a pair of earrings

Note : With a pair of ……. A singular verb will be used.

A pair of trousers was lying on the bed.

RULE 20

These nouns appear to be plural but are actually singular and we use a singular verb with them.

News, Innings, Politics, Summons, linguistics.

Names of subjects ending in s.

Examples -Physics, Economics, Ethics, Mathematics,

Names of diseases ending in s.

Examples – Mumps, Measles, Rickets Shingles, Billiards, Athletics etc

Examples

Athletics is good for young people.

Linguistics is the study of language.

Darts is a popular game in England.

Billiards is played all over the world.

RULE 21

These nouns appear as singular but are plural and a plural verb is used with them.

Cattle, cavalry, infantry, poultry, peasantry, children , gentry, police, people, etc.

RULE 22

These noun are used in singular form only and they are uncountable form only.

Scenery, Poetry, Furniture, Advice, Information, Hair, Business, Mischief, Bread, Stationery, Crockery, Luggage, Baggage, Postage, Knowledge, Wastage, Money, Jewellery, Breakage, Equipment, Work,

Evidence, Word ,Fuel .

We cannot use a/an with these nouns immediately before them. To express a quantity of an uncountable noun, use a word or expression like some, a lot of, much, a bit of, a great deal of.

Examples.

Page 27: JSpiders - Aptitude

www.Jspiders.com www.facebook.com/JSpiders.Basavanagudi/ Mb.9686114422 www.qspiders.com twitter.com/JBasavanagudi

There has been a lot of research into the causes of this disease.

He gave me a great deal of advice before my interview.

Can you give me some information about uncountable nouns?

He did not have much sugar left.

A lot of wonderful jewellery were available in the shop.

A piece of jewellery is here for you.

Note: The verb used varies as per the usage of the noun.

Rule 23

Hair: Be careful with the noun hair which is normally

uncountable in English, so it is not used in the plural. It can be countable only when referring to individual hairs.

Example : She has long thick hair.

Two strands of grey hair were found by the police in the car.

Paper: When paper refers to exams it is countable

otherwise it is singular.

A lot of paper is used by newspapers.

I have to answer two English papers in May.

RULE 24

Deer, sheep, series, species, fish, crew, team, jury, aircraft, counsel etc. These words are in the same form in singular and plural.

SPOTTING THE ERROR

1. a) Neither of them/b) are going to attend/c) the

party on 10th October/d) no error

2. a) He walked five miles which are really a great

distance/b) for a man like him/c) who is not only old

but also ill./d) no error

3. a) The rise and fall of the/b) of the tide are due/c) to

lunar influence./d) no error

4. a) the newer type of automatic/b) machine wash/c)

clothes faster./d) no error

5. a)The secretary and principal of the college/b) are

attending/c) the council meeting at the

collectorate./d) no error.

6. a) It is I/b) who is to blame/c) for this bad

situation./d) no error.

7. a) Patience as well as perseverance/b) are

necessary/c) for success./d) no error

8. a) One of my desires/b) are to become/c) a

doctor./d) no error.

9. a) Neither of them/b) sent their papers/c)in time for

the last seminar/d) no error.

10. a) He is/b) one of the tallest boy/c) in the class./d)

no error

11. a) Many a boy/b) have not done their/c)

homework properly./d) no error

12. a) Many a men/b) attended the meeting/c) last

night./d) no error

13. a) Either you/b) or he/c) are happy./d) no error

3. VOICE

Active voice:- When the person or the thing in the

subject does something is called active voice; means

the subject is in action.

Passive voice:- When something is done to the subject ;

means the person or the thing of the subject does not

perform the action denoted by the verb is called

passive voice.

e.g.:- Ranjan writes a letter.

A letter is written by Ranjan.

Who did this? (Active voice)

By whom was this done?

Conversion of Active to Passive Sentence in different Tenses

1. Simple Present

Active voice: Subject+V1+Object

Passive Voice: Object +is/am/are+V3 + Subject

e.g. Active voice: He sings a song.

Passive voice: A song is sung by him.

2. Present Continuous

Active voice: Subject +is/am/are+V1+ing+Object

Passive Voice: Object

+is/am/are+being+V3+by+Subject

e.g. Active voice: I am writing a letter.

Passive voice: A letter is being written by me.

3. Present Perfect

Active voice: Subject +has/have+V3+Object

Passive Voice: Object+

has/have+been+V3+by+Subject

e.g. Active voice: She has finished his work.

Passive voice: His work has been finished by her.

4. Present perfect continuous tense:

Passive form:object+have/has+been+being+v3

Active voice: They have been doing the reseaechfor

years.

Passive voice: The research has been being done for

years.

5. Simple Past

Active voice: Subject+V2+Object

Passive Voice: Object +was/were+V3+by+Subject

e.g.: Active voice: I killed a snake.

Passive voice: A snake was killed by me.

6. Past Continuous

Active voice: Subject +was/were+V1+ing+Object

Passive Voice: Object

+was/were+being+V3+by+Subject

e.g.: Active voice: He was driving a car.

Passive voice: A car was being driven by him.

Page 28: JSpiders - Aptitude

www.Jspiders.com www.facebook.com/JSpiders.Basavanagudi/ Mb.9686114422 www.qspiders.com twitter.com/JBasavanagudi

7. Past Perfect

Active voice: Subject+Had+V3+Object

Passive Voice: Object+Had+been+V3+by+Subject

e.g: Active voice: They had completed the assignment.

Passive voice: The assignment had been completed by

them.

8. Past Perfect Continuous Tense

Passive form:had been being+v3

Active voice: The police had been following him for

years.

Passive voice: He had been being followed for years

9. Simple Future

Active voice: Subject +Will/Shall+V1+Object

Passive Voice: Object

+Will/Shall+Be+V3+By+Subject

Active voice: She will buy a car.

Passive voice: A car will be bought by her.

10. Future Continuous

Active voice: Subject

+Will/Shall+be+V1+ing+Object

Passive Voice: object+shall/will+be+being+v3

e.g.: active: I shall be doing this.

Passive: this will be being done by me.

11. Future Perfect

Active voice: Subject +Shall/will+have+V3+Object

Passive Voice: Object

+Shall/will+have+been+V3+by+Subject

Active voice: You will have started the job.

Passive voice: The job will have been started by you.

12. Future perfect continuous tense:

Passive form: shall/will+have been being+v3

Active: we shall have been building the fort for five

years.

Passive: the fort will have been being built for five

years.

Modal verbs: shall, will, can, could, may, might,

should, would, must, ought are called modal

Auxiliaries.

Passive form: modal auxiliary+be+v3

Active: They will kill the tiger.

Passive: The tiger will be killed

Active Voice: The Mason Is building the wall.

Passive Voice: The wall is being built by the Mason.

Active Voice: He will finish The work in a fortnight.

Passive Voice: The work will be finished by him in

a fortnight.

Active Voice: Who did this?

Passive Voice: By whom was this done?

Active Voice: The cat killed the mouse.

Passive Voice: the mouse was killed by cat.

The Fire engulfed the office.(Active voice)

The office was engulfed in by the fire.(Passive

Voice)

Who wrote this book? (Active voice)

By whom was this book written? (Passive voice)

Why did your brother write such a letter? (Active

Voice)

Why was such a letter written by your brother?

(Passive Voice)

Shut The door.(Active voice)

Let the door be shut (Passive Voice)

Help the elders.(Active voice)

The elders should be helped.(Passive Voice)

Close the door.(AV)

Let the door be closed.(PV)

Do not disturb him.(AV)

Let him not be disturbed.(PV)

Do not tell the truth.(AV)

Let not the truth be told.(PV)

Will the government build a bridge?(AV)

Will a bridge be built by the government?(PV)

1. Question: He opens the door.

Answer: The door is opened by him.

2. Question: I am going to bring him here.

Answer: He is going to be brought here by me.

3. Question: They believe Veerappan to be

dangerous.

Answer: Veerappan is believed to be dangerous.

4. Question: Let him be told to do it.

Answer: Tell him to do it.

5. Question: Your promises are to be kept by you.

Answer: You are to keep your promises.

6. Question: Many types of fish were being sold by

then at the market.

Answer: They were selling many types of fish at the

market.

7. Question: We have cycled more than ten

kilometre.

Answer: More than ten kilometre have been cycled

by us.

8. Question: Please see that the room is swept and

mopped before I return.

Answer: Please see that somebody sweeps and

mops the room before I return.

9. Question: Let the window be opened.

Answer: Open the window.

10. Question: Many factors within our nation affect

the economy.

Answer: The economy is affected by many factors

within our nation.

11. Question: I know that he did the work.

Answer: That the work was done by him is known

to me.

12. Question: He was caught by the police for

littering up the streets.

Answer: The police caught him for littering up the

street.

13. Question: The task should have been completed

by the student.

Answer: The student should have complete the

task.

14. Question: The painting would have been stolen

if he hadn’t helped.

Answer: Someone would have stolen the painting if

he hadn’t helped.

Page 29: JSpiders - Aptitude

www.Jspiders.com www.facebook.com/JSpiders.Basavanagudi/ Mb.9686114422 www.qspiders.com twitter.com/JBasavanagudi

15. Question: Parents love their children.

Answer: Children are loved by their parents.

16. Question: The lady is treading on the flowers.

Answer: The flowers are being trod on by the lady.

17. Question: The historical society built the

museum.

Answer: The museum was built by the historical

society.

18. Question: My last venture undid me

completely.

Answer: I was completely undone by my last

venture.

19. Question: He has sent me a present.

Answer: A present has been sent to me by him.

20. Question: The child upset the salt over

everything.

Answer: The salt was upset over everything by the

child.

21. Question: It is time to water the plants.

Answer: It is time for the plants to be watered.

22. Question: Shut the door.

Answer: Let the door be shut.

23. Question: Did he plan an excursion to

mountains?

Answer: Was an excursion to mountains planned by

him?

24. Question: Our vehicle had been stolen from our

garage by the same burglary team.

Answer: The same burglary team had stolen our

vehicle from our garage.

25. Question: We shall write a novel.

Answer: A novel will be written by us.

26. Question: I shall be obliged to go.

Answer: Circumstances will oblige me to go.

27. Question: I didn’t speak even a single word in the

meeting.

Answer: Even a single word was not spoken by me in

the meeting.

28. Question: The situation could hardly have been

altered by the interference of the police force.

Answer: The interference of the police force could

hardly have altered the situation.

29. Question: Rajesh could not complete his paper

because he had a headache.

Answer: The paper could not be completed by Rajesh

because he had a headache.

30. Question: Who tore the curtains yesterday?

Answer: By whom were the curtains teared yesterday?

31. Question: These days most of the students do their

assignments with the help of the internet.

Answer: These days the assignments are done with the

help of the internet by most of the students.

32. Question: Who had laughed at you?

Answer: By whom had you been laughed at?

33. Question: Who gave you the money?

Answer: By whom were you given the money?

34. Question: Ratan is performing an experiment.

Answer: An experiment is being performed by Ratan.

35. Question: That too many cooks spoil the broth is

known to all.

Answer: All know that too many cooks spoil the broth.

36. Question: The judge ordered the murderer to be

hanged.

Answer: It was ordered by the judge to hang the

murderer.

37. Question: Kindly offer your remarks.

Answer: You are requested to offer your remarks.

38. Question: We prohibit smoking.

Answer: Smoking is prohibited.

39. Question: The Swiss regarded him as an imposter

and called him a villain.

Answer: He was regarded as an imposter and was

called a villain by the Swiss.

40. Question: Sita learns her lessons daily.

Answer: Her lessons are learnt daily by Sita.

4. NARRATION

Direct Speech: If a sentence is directly spoken by speaker

is called direct speech.

Example: Manish Says, “she is very honest”.

NOTE: Direct Speech always comes under inverted

Commas(“”)

Indirect Speech: If a sentence is changed by some person

with set of rules and presented is call indirect speech.

Example: Manish says that she is very honest.

NOTE: Indirect Speech does not come under inverted

Commas (“”)

Following Set Of Rules Are Used While Converting

Direct To Indirect Speech:

1st person (I, we)>>>>>>>>according to subject

2nd person (you)>>>>>>>>according to object

3rd person (he, she, they, it)>>>>>>>No change

Examples:

She says,” I work hard”.

She says that she works hard. (Indirect Speech)

In above sentence ‘I’ has been changed in ‘She’

according to subject.

He tells me that he works hard. (Indirect Speech)

He says to me, “He works hard.”

In above sentence ‘he’ is 3rd person and no change.

Page 30: JSpiders - Aptitude

www.Jspiders.com www.facebook.com/JSpiders.Basavanagudi/ Mb.9686114422 www.qspiders.com twitter.com/JBasavanagudi

Examples:

She says,” I work hard”.

She says that she works hard. (Indirect Speech)

In above sentence ‘I’ has been changed in ‘She’

according to subject.

He tells me that he works hard. (Indirect Speech)

He says to me, “He works hard.”

In above sentence ‘he’ is 3rd person and no change.

Few Things to Note:

1. ‘Says to ‘ changes to ‘tells’ and ‘said to ‘ changes to ‘told’.

2. Object need to come after tell, told, requested, ordered

and forbade.

3. If sentence is in ‘present or future tense’ then no need to

change tense of indirect speech.

Examples: Ravi Says,” It’s raining.”

Ravi says that it’s raining (Indirect Speech)

While changing direct to indirect speech, Following

is change in tense:

1. Present Tense:

1. Simple Present will be changed in Simple Past i.e.’

Do/Does’ will be converted into ‘Did’.

2. Present Continuous will be changed in Past continuous

i.e. is/am/are will be converted into was/were.

3. Present Perfect /Present Perfect continuous will be

changed in Past Perfect/Past Perfect Continuous.

i.e. has /have will be converted into ‘had’.

2. Past Tense:

1. Simple Past will be changed in Past Perfect i.e. did will be

converted into Had+Verb 3

2. Past Continuous will be changed in Past perfect

Continuous i.e. was /were will be converted into had been

3. Past Perfect /Past Perfect Continuous: NO Change

3. Special Cases:

If Reporting verb is in past and Speech is ‘Idiom

/Phrases/universal Truth/Any historical facts then No need

to change tense of reported speech.

Example:

Manoj Said,” Sun rises from East.”

Manoj Said that Sum rises from East. (Indirect Speech)

Will →Would,Shall→Should,May→Might,Can →could

today→that day,tomorrow→the next day, yesterday→the

previous day,tonight→that night,

Last→previous,here→there,this →that,these →those,ago

→before,now→then

Examples:

The teacher Said to the student,” why were you absent

yesterday?

The teacher asked student why he has been absent the

previous day. (Indirect Speech)

Exclamatory Sentences, Following changes need to

done:

Alas! →exclaimed with sorrow

Oh! →exclaimed with regret

Wow! →Exclaimed with Joy

Hurray! →Exclaimed with Joy

Ugh! →Exclaimed with despise

Page 31: JSpiders - Aptitude

www.Jspiders.com www.facebook.com/JSpiders.Basavanagudi/ Mb.9686114422 www.qspiders.com twitter.com/JBasavanagudi

Examples:

He said,”Fi! And Vomited.

He exclaimed with disgust and vomited

Rules for Converting Interrogative sentences (WH

Family):

1. ‘Said to’ will be changed into asked, questioned or

inquired.

2. Helping verb will be used after subject.

3. Question mark (?) will be changed in full stop (.)

4. Rest Changes will according to Rules.

Examples:

He said to me, “when will you come back?”

He asked me when I would come back. (Indirect Speech)

My father said to me,” when will your college start?”

My father asked me when my college would start. (Indirect

Speech)

He Said to him,” Did you intend to come with me?”

He asked him if he had intended to come with him. (Indirect

Speech)

NOTE: If Question starts with WH family then no

need to use conjunction and if Question start with

helping verb i.e. do, does, did then we need to use

if/whether to connect sentences.

Exercises:

‘Did you come here yesterday?’ the man asked the boy.

‘Have you ever been to a zoo?’ Mr. James asked me.

‘Did you have any difficulty in finding my address?’ the old

woman asked me.

‘Has your father gone to work?’ I asked the boy.

‘May I go home?’ the boy asked.

‘Shall we invite John to the party?’ Mark asked.

Sony asked Pawan, ‘Have you seen my new car?’

‘Are you a clever boy?’ the stranger asked Harry.

Answers:

The man asked the boy if he had gone there yesterday.

Mr.James asked me if I had ever been to a zoo.

The old woman asked me if I had any difficulty in finding

her address.

I asked the boy if his father had gone to work.

The boy asked if he might go home.

Mark asked if they should invite John to the party.

Sony asked Pawan if he had seen his new car.

The stranger asked Harry whether he was a clever boy.

Last Years /Previous Years Narration asked in

competitive examination

“could i have a cup of coffee”, she said.

She asked if she could have a cup of coffee.

‘Please,please don’t do anything dangerous’,said his wife.

His wife begged him not to do anything dangerous.

He said to his friends,”Let us play some country music and

dance”

He suggested to his friends that they could play some

country music and dance.

“Which way is the post office?” the lady asked

The lady asked the way to the post office.

” Do you know the way to the bus station?” Ram said to

Mahim.

Ram asked Mahim if he knew the way to the bus station.

‘Please be patient and letter will come’,said the teacher to

student.

The teacher advised the student to be patient and wait for

letter.

Page 32: JSpiders - Aptitude

www.Jspiders.com www.facebook.com/JSpiders.Basavanagudi/ Mb.9686114422 www.qspiders.com twitter.com/JBasavanagudi

“I must hurry,my father is always furious if any one of us is

late for

meals”,she said

She said that she must hurry as her father was always

furious if any one of them

was late for meals.

He said,”As your mother is ill, you must leave at once.”

He told him that as his mother was ill , he should leave at

once.

She Shouted,” I am going home”.

She shouted to them that she was going home.

Jennifer said,” What will you do when you leave school?”

Jennifer asked someone what she would do when she would

leave school.

Mr Pritham said,” I shall go to Delhi tomorrow.”

Mr Pritham said that he would go to Delhi the next day.

Next morning at breakfast Ramesh’wife said to

him,”Ramesh I think I can tell what is amiss with our clock.”

Next morning at breakfast Ramesh’s wife addressed him by

name and told him that she thought that she could tell what

was amiss with their clock.

“Are you the manager?” I said

I asked him whether he was the manager.

The minister announced that a new employment scheme

would be implemented from

the following month.

The minister said,”A new employment scheme will be

implemented next month

onwards.

“Have you finished your work,Ahmad?”asked Vinod.

Vinod asked Ahmad whether he had finished his work.

Sachin said,”could you please write the project for

me,Shweta?”

Sachin requested Shweta to write the project for her.

“It’s a cold day today !’Cried the boy.

The boy exclaimed that it was cold that day.

“Are your examinations over?”asked the teacher.

The teacher asked whether his examination were over.

“Do your duty.”,the father told his son.

The father advised his son to do his duty.

Mike said ,”Don’t fly via Paris.”

Mike advised me not to fly via Paris.

Jane said to me,”Do you want to dance?”

Jane asked me if I wanted to dance.

Saint George,strike for us!” exclaimed the Knight.

The Knight prayed that Saint George might strike for them.

“I usually take my dog out for a walk when I come home

from work”,he said.

He said that he usually took hos dog out for a walk when he

came home from work.

The Officer said,”Cowards ! you shall soon be put to death.”

The Officer called them cowards and said that they would

soon be put to death

Rajiv said to me,”He plays with right hand.”

Rajiv told me that he plays with right hand.

My father said to me,”will you sit and study at least now.”

My father asked me if i would sit and study at least then.

The guide said to the swimmer,”Don’t swim out too far.”

The guide forbade the swimmer to swim out too far.

Page 33: JSpiders - Aptitude

www.Jspiders.com www.facebook.com/JSpiders.Basavanagudi/ Mb.9686114422 www.qspiders.com twitter.com/JBasavanagudi

He Said to me,”where are you going?”

He asked me where I was going.

He wished me respectfully and told that he had come to join

the office.

He said to me,”Good morning Sir ! I have come to join the

office.”

He said to his father,”Shall I go to market now?”

He asked his father if he should go to market then.

“Oh! how foolish I have been in money matters!”

He confessed regretfuly that he had been very foolish in

money matters.

I said,”Good-bye my beloved friends.”

I bade me beloved friends good-bye.

The Commander said,”Let the gates be left open.”

The Commander ordered for the gates to be left open.

I said to my mother,”Please warm this milk.Don’t boil it.”

I requested my mother to warm that milk and asked her not

boil it.

I said to my friend,”Can i borrow your dictionary for one day

only?”

I asked my friend if i could borrow his dictionary for one day

only.

He asked me if I would like to have lunch with him that day.

He said to me,”Would you like to have lunch with me

today?”

“Sam,Will you help me carry my bags?” asked the old

woman.

The old woman asked Sam whether he would help her carry

her bags.

The teacher said to me,”I hope you will bring credit to my

school.”

The teacher hoped that I would bring credit to his school.

“What an excellent peice of art!”she said.

She exclaimed with appreciation that it was an excellent

piece of art.

“they are late.” she has already told us.

She has already told us that they are late.

The Teacher said.”Don’t come to school on Monday because

it is a holiday..”

The teacher told us not to come to school on Monday

because it is a holiday.

The teacher said to his student,”Do you have any excuse for

coming late to school today?”

The teacher asked his student whether he had any excuse for

coming late to the school that day.

5. CONDITIONAL SENTENCE

When two actions take place one after another, it is

called a conditional sentence.

e.g. You will succeed provided you work hard.

Look at the following conditionals:

1. If 2. Provided 3. As soon as

4. No sooner than 5. Unless 6. Until

7. When

There are two kinds of conditionals.

Real

Unreal

Real conditionals are used for possible situations.

The present tense is used in the if - clause (or

conditional clause), and the future tense is used in the

result clause.

If he comes to the library ,I will give him these books.

Unreal conditions are used for impossible or unreal

situations.

If Clause Main Clause

If+ present………………………future(Result)

If+ past………………………………would+ verb (Result)

Page 34: JSpiders - Aptitude

www.Jspiders.com www.facebook.com/JSpiders.Basavanagudi/ Mb.9686114422 www.qspiders.com twitter.com/JBasavanagudi

If +past perfect……………………would +have+ past

participle(result)

e.g.: if he were rich, he would go the USA for higher

studies.

If I were a bird, I would fly in the sky.

If I win the contest, I will buy a new house.

If I were there, I would make a speech.

If I had been there, I would have made a speech.

I will help him, provided he mends his ways.

Unless he takes care of his health, he will not recover.

If you have finished the work, you may leave.

If I had money , I would lend it to you.

If I had seen you, I could have stopped my car.

Had I seen you, I would have stopped my car.

He scolded me as if he were my father.

I shall support him so long as I am alive.

As soon as the train comes, the platform will get

overcrowded.

When he comes to delhi I will go to meet him.

SPOTTING THE ERROR

1. a) If he came to me/b) I would have given/c) my car./d) no

error.

2. a) Had he invited me/b) I would have attended/c) the

function/d) no error

3. a) If he will work hard/b) he will surely/c) get the job of

his choice./d) no error.

4. a) Unless I do not /b) see his ticket/c)I will not let him

sit here./d) no error.

5. a) We will know the truth/b) after/c) the investigation

finished/d) no error.

6. a) Until the train will not get the signal,/b) it will not /c)

leave the platform./d) no error.

7. a) If I was you/b) I would not tolerate him/c) for a

moment./d) no error.

8. a) But for his help/b) the patient would /c) have

died./d) no error

9. a) But for his prompt action/b) many people would/c)

have lost their savings/d) no error.

10. a) I will meet him/b) when he/c) will come./d) no error.

11. a) Had I realized/b) your house was such a long way

off,/c) I would take a taxi./d) no error.

REASONING

1. COADING-DECODING

1. In a certain code language RUSTICATE is written as

QTTUIDBSD. How would be STATISTIC be written in

that code:

a) RSBUJTUHB b) RSBUITUHB

c) RSBIJHTUH d) RSHUITUHB

2. In a certain code language BEAT is written as YVZG, then what

will be the code of MILD:

a) ONRW b) NOWR

c) ONWR d) NROW

3. In a certain code language, the words COME AT ONCE is

written as XLNVZGLMXV. In the same code, which of the

following would be OK:

a) KL b) LM

c) KM d) LP

4. In a certain language FLOWER is written as UOLDVI, then how

Is TERMINAL written in that code:

a) FLKPMROZ b) GVINRMZO

c) RVNIGLKA d) MNIVGYEO

5. If the word EARTH be written as QPMZS in coded form, how

can HEART be written following the same code:

a) SQPZM b) SQMPZ

c) SPQZM d)SQPMZ

6. In a certain code, BAT=23 and CAT=24, then how will

you code BALL:

a) 27 b) 28

c) 32 d) 120

7. If A=26, SUN=27, then CAT=

a) 24 b) 27

c) 57 d) 58

8. DEF, DEF2, DE2F2, _____, D2E2F3

a) DEF3 b) D3EF3

c) D2E3F d) D2E2F2

9. In a certain code, RAIN is written as 8$%6and MORE is

written as 7#8@. How is REMAIN written in that code:

a) #@$7%6 b) #@&$%6

c) 7@$#6& d) 8@7$%6

10. If ‘paper’ is called ‘wood’, ‘wood’ is called ‘straw’,

‘straw’ is called ‘grass’, ‘grass’ is called ‘rubber’ and

‘rubber’ is called ‘cloth’, what is the furniture made up

of:

a) Paper b) wood

c) Straw d) grass

11. If ‘blue’ means ‘green’ ,’green’ means ‘white’, ‘white’

means ‘yellow’, ‘yellow’ means ‘black’, ‘black’ means

‘red’ and ‘red’ means ‘brown’, then what is the colour

of milk:

a) Black b) brown

c) Blue d) yellow

In a certain code language,

i. ‘Pit na sa’ means ‘you are welcome’

ii. ‘na ho pa la’ means ‘they are very good’

iii. ‘ka da la’ means ‘who is good’

iv. ‘od ho pit la’ means ‘they welcome good people;

12. Which of the following means ‘people’

a) Od b) la

b) Ho d) pit

13. Which of the following means ‘very’ in code language:

a) Na b) da

Page 35: JSpiders - Aptitude

www.Jspiders.com www.facebook.com/JSpiders.Basavanagudi/ Mb.9686114422 www.qspiders.com twitter.com/JBasavanagudi

c) Pa d) none

In a certain language

i. ‘pit dar na’ means ‘you are good’

ii. ‘dar tok pa’ means ‘good and bad’

iii. ‘tim na tok’ means ‘they are bad’

14. Which word stand for ‘they’

a) Na b) tok

c) Tim d) pit

15. Which word stand for ‘good’

a) Tim b)pit

c) Dar d) na

2. SYLLOGISM

In each of the following questions two statements are

given and these statements are followed by two

conclusions numbered (1) and (2). You have to take

the given two statements to be true even if they seem

to be at variance from commonly known facts. Read

the conclusions and then decide which of the given

conclusions logically follows from the two given

statements, disregarding commonly known facts.

Give answer:

(A) If only (1) conclusion follows

(B) If sonly (2) conclusion follows

(C) If either (1) or (2) follows

(D) If neither (1) nor (2) follows and

(E) If both (1) and (2) follow.

1. Statements: Some actors are singers. All the singers are dancers.

Conclusions:

i. Some actors are dancers.

ii. No singer is actor. 2. Statements: All the harmoniums are

instruments. All the instruments are flutes.

Conclusions:

1. All the flutes are instruments.

2. All the harmoniums are flutes.

3. Statements: Some mangoes are yellow. Some oranges are mangoes.

Conclusions:

1. Some mangoes are green.

2. oranges are yellow.

4. statement: All the locks are keys. All the keys are

bats. Some watches are bats.

Conclusions: 1. Some bats are locks.

2.Some watches are keys.

3. All the keys are locks.

a) only 1 and 3

b) only 1

c) only 2

d) only 1 and 3

5. statements: Some keys are staplers. Some

staplers are stickers. All the stickers are pens.

Conclusions:a1. Some pens are staplers.

2.Some stickers are keys.

3. No sticker is key.

4. Some staplers are keys.

a) Only 1 and 2

b) Only 2 and 4

c) Only 2 and 3

d) Only 1 and 4 and either 2 or 3

3. BLOOD RELATIONS

1. Pointing towards a person , a man said to a eoman,

”his mother is the only daughter of your father.” How

is the woman related to that person:

a) Daughter b)sister

c) Mother d) wife

2. Pointing to a lady in photograph, shaloo said “her

son’s father is the son-in-law of my mother.” How is

shaloo related to the lady:

a) Aunt b) sister

c) Mother d) cousin

3. In a joint family, there are father, mother, 3 married

sons, and one unmarried daughter. Of the sons, two

have 2 daughters each and one has a son. How many

female members are there in the family:

a) 2 b) 3

c) 6 d) 9

A+B means A is the son of B

A-B means A is the wife of B

A×B means A is the brother of B

A÷B means A is the mother of B and

A=B means A is the sister of B

4. What does P+R-Q mean?

a) Q is the father of P

b) Q is the son of P

c) Q is the uncle of P

d) Q is the brother of P

5. What does P×R ÷Q mean:

a) P is the brother of Q

b) P is the father of Q

c) P is the uncle of Q

d) P is the nephew of Q

6. What does P=R+Q mean:

a) P is the aunt of Q

Page 36: JSpiders - Aptitude

www.Jspiders.com www.facebook.com/JSpiders.Basavanagudi/ Mb.9686114422 www.qspiders.com twitter.com/JBasavanagudi

b) P is the daughter of Q

c) P is the niece of Q

d) P is the sister of Q

7. What does p=R÷Q mean:

a) P is the aunt of Q

b) P is the sister of Q

c) Q is the niece of P

d) Q is the daughter of P

8. Pointing to a photograph a man said,”I have no brother

or sister but the man’s father is my father’s son.”

Whose photograph was it?

a) His own b) his son’s

c) His nephew’s d) his father’s

4. DIRECTION SENSE

1. One morning Udai and Vishal were talking to each

other face to face at a crossing. If Vishal's shadow

was exactly to the left of Udai, which direction was

Udai facing?

a) North-East b) north-

west

c) South-east d) south-west 2. If South-East becomes North, North-East

becomes West and so on. What will West

become?

a) West b) south

c) North-east d) south-west

3. A man walks 5 km toward south and then turns to

the right. After walking 3 km he turns to the left

and walks 5 km. Now in which direction is he from

the starting place?

a) West b) south

c) North-East d) north-west

4. Rasik walked 20 m towards north. Then he

turned right and walks 30 m. Then he turns

right and walks 35 m. Then he turns left and

walks 15 m. Finally he turns left and walks 15

m. In which direction and how many metres

is he from the starting position?

a) 15 m West b) 30 m East

c) 30 m West d) 45 m East

5. Starting from the point X, Jayant walked 15

m towards west. He turned left and walked 20 m. He then turned left and walked 15 m. After this he turned to his right and walked 12 m. How far and in which directions is now Jayant from X?

a) 32 m, South b) 47 m, East

c) 42 m, North d) 27 m, South

6. A boy rode his bicycle Northward, then

turned left and rode 1 km and again turned left and rode 2 km. He found himself 1 km

west of his starting point. How far did he ride northward initially?

a) 1km b) 2km

c) 3km d) 5km

5. SEATING ARRANGEMENT

1. A, P, R, X, S and Z are sitting in a row. S and Z are in the centre. A and P are at the ends. R is sitting to the left of A. Who is to the right of P ? a) A b) X c) S d)Z

2. A, B, C, D and E are sitting on a bench. A is

sitting next to B, C is sitting next to D, D is not sitting with E who is on the left end of the bench. C is on the second position from the right. A is to

the right of B and E. A and C are sitting together. In which position A is sitting ? a) Between B and D b) between B and C c) Between E and D d) ) between E and C

P, Q, R, S, T, U, V and W are sitting round the circle and are facing the centre:

P is second to the right of T who is the neighbour of R and V.

S is not the neighbour of P. V is the neighbour of U. Q is not between S and W. W is not between U and S.

3. Which two of the following are not neighbours

?

a) RV b) UV c)RP d) QW

4. Which one is immediate right to the V ?

a) P b) U c) R d)T

5. Which of the following is correct ?

a) P is to the immediate right of Q

b) R is between U and V

c) Q is to the immediate left of W

d) U is between W and S

6. What is the position of S ?

a) Between U and V

b) Second to the right of P

c) To the immediate right of W

d) Data inadequate.

Five girls are sitting on a bench to be

photographed. Seema is to the left of Rani and

to the right of Bindu. Mary is to the right of

Rani. Reeta is between Rani and Mary.

7. Who is sitting immediate right to Reeta ?

Page 37: JSpiders - Aptitude

www.Jspiders.com www.facebook.com/JSpiders.Basavanagudi/ Mb.9686114422 www.qspiders.com twitter.com/JBasavanagudi

a) Bindu b) rani

c) Mary d) sita 8. Who is in the middle of the photograph ?

a) Bindu b) rani c) rita d) seema 9. Who is second from the right ?

a) Marry b) rani c) rita d) bindu 10. Who is second from the left in

photograph ?

a) Rita b) marry c) bindu d) seema

I. A,B,C,D,E,F and G are sitting in a circle facing the centre and playing cards.

II. E is neighbour of B and D III. G is not between F and C

IV. F is to the immediate right of A 11. Which pair of person is not sitting adjacent

to each other: a) BA b) CB c) DE d) GD

12. Which of the following pair has the second person sitting immediately to the right of the first:

a) AB b) CB c) EA d) DG e) none

13. What is the position of F? a) Third to the left of C b) Second to the right of C c) Immediate left of A d) Immediate right of B

14. Who are the neighbours of B?

a) AF b) CD c) FC d) none

6. RANKING

1. Rohan ranks seventh from the top and twenty-

sixth from the bottom in a class. How many students are there in the classs.

a) 31 b) 32 c) 33 d) 34 2. Manic is fourteenth from the right end in a

row of 40 boys. What is his position from left

end? a) 24th b) 25th c) 26th d) 27th

3. In a row of boys facing north, A is sixteenth from the left end and C is sixteenth from the right end. B who is fourth to the right of A, is fifth to the left of C in the row.how many boys are there in the row: a) 39 b) 40 c) 41 d) 42

4. In a row of 40 girls, when komal was shifted

to her left by 4 places, her number from the left end of the row becomes 10. What wqas the number of swathi from the right end of the row if swati was three places to the right of komal’s original position: a) 22 b) 23 c) 25 d) 26

5. Aman is 16th from the left end in a row of boys and vivek is 18th from the right end .

gagan is 11th from aman towards the right and 3rd from vivek towards the right end . how many boys are there in the row: a) 40 b) 41 c) 42 d) 48

6. Nitin ranks eighteenth in a class of 49

students. What is his rank from the last a) 18 b) 19 c)31 d) 32

7. Sam ranked ninth from the top and thirty eighth from the bottom in a class. How many students are there in the class? a) 45 b) 46 c) 47 d) 48

8. A boy in a row is nineteenth in order from

both the ends. How many boys are there in the class: a) 27 b) 37 c) 38 d) 39

9. Bharti is 8 ranks ahead of Divya who ranks

twenty sixth in a class of 42. What is bharti’s rank from the last: a) 9th b) 24th c) 25th d) 34th

10. In a class of 60, where girls are twice that of boys, kamal ranked seventeenth from the top. If there are 9 girls ahead of kamal, how many boys are after him in rank: a) 3 b) 7 c) 12 d) 23